Sei sulla pagina 1di 109

INTRODUCTION TO DYNAMICS

We shall analyze the motion of systems of particles and rigid bodies that are undergoing translational
and rotational motion about a fixed direction. Because the body is translating, the axis of rotation is no
longer fixed in space. We shall describe the motion by a translation of the center of mass and a rotation
about the center of mass. By choosing a reference frame moving with the center of mass, we can
analyze the rotational motion separately and discover that the torque about the center of mass is equal
to the change in the angular momentum about the center of mass. For a rigid body undergoing fixed axis
rotation about the center of mass, our rotational equation of motion is similar to one we have already
encountered for fixed axis rotation,

Dynamics is the branch of classical mechanics concerned with the study of forces and their effects on
motion. Isaac Newton defined the fundamental physical laws which govern dynamics in physics,
especially his second law of motion.

Galileo was instrumental in establishing observation as the absolute determinant of truth, rather than
“logical” argument. Galileo’s use of the telescope was his most notable achievement in demonstrating
the importance of observation. He discovered moons orbiting Jupiter and made other observations that
were inconsistent with certain ancient ideas and religious dogma. For this reason, and because of the
manner in which he dealt with those in authority, Galileo was tried by the Inquisition and punished. He
spent the final years of his life under a form of house arrest. Because others before Galileo had also
made discoveries by observing the nature of the universe, and because repeated observations verified
those of Galileo, his work could not be suppressed or denied. After his death, his work was verified by
others, and his ideas were eventually accepted by the church and scientific communities.

Galileo also contributed to the formation of what is now called Newton’s first law of motion. Newton
made use of the work of his predecessors, which enabled him to develop laws of motion, discover the
law of gravity, invent calculus, and make great contributions to the theories of light and color. It is
amazing that many of these developments were made with Newton working alone, without the benefit
of the usual interactions that take place among scientists today.

It was not until the advent of modern physics early in the 20th century that it was discovered that
Newton’s laws of motion produce a good approximation to motion only when the objects are moving at
speeds much, much less than the speed of light and when those objects are larger than the size of most
molecules (about {\text{10}}^{-9}\phantom{\rule{0.25em}{0ex}}\text{m} in diameter). These constraints
define the realm of classical mechanics, as discussed in Introduction to the Nature of Science and
Physics. At the beginning of the 20th century, Albert Einstein (1879–1955) developed the theory of
relativity and, along with many other scientists, developed quantum theory. This theory does not have
the constraints present in classical physics. All of the situations we consider in this chapter, and all those
preceding the introduction of relativity in Special Relativity, are in the realm of classical physics.
Dynamics

Branch of physical science and subdivision of mechanics that is concerned with the motion of material
objects in relation to the physical factors that affect them: force, mass, momentum, energy.

A brief treatment of dynamics follows. For full treatment, see mechanics.

Dynamics can be subdivided into kinematics, which describes motion, without regard to its causes, in
terms of position, velocity, and acceleration; and kinetics, which is concerned with the effect of forces
and torques on the motion of bodies having mass. The foundations of dynamics were laid at the end of
the 16th century by Galileo Galilei who, by experimenting with a smooth ball rolling down an inclined
plane, derived the law of motion for falling bodies; he was also the first to recognize that force is the
cause of changes in the velocity of a body, a fact formulated by Isaac Newton in the 17th century in his
second law of motion. This law states that the force acting on a body is equal to the rate of change of
the body’s momentum. See mechanics; Newton’s laws of motion

Newton’s Law of Motion

Sir Isaac Newton's three laws of motion describe the motion of massive bodies and how they interact.
While Newton's laws may seem obvious to us today, more than three centuries ago they were
considered revolutionary.

Newton was one of the most influential scientists of all time. His ideas became the basis for modern
physics. He built upon ideas put forth from the works of previous scientists including Galileo and
Aristotle and was able to prove some ideas that had only been theories in the past. He studied optics,
astronomy and math — he invented calculus. (German mathematician Gottfried Leibniz is also credited
with developing it independently at about the same time.)

Newton is perhaps best known for his work in studying gravity and the motion of planets. Urged on by
astronomer Edmond Halley after admitting he had lost his proof of elliptical orbits a few years prior,
Newton published his laws in 1687, in his seminal work "Philosophiæ Naturalis Principia Mathematica"
(Mathematical Principles of Natural Philosophy) in which he formalized the description of how massive
bodies move under the influence of external forces.

In formulating his three laws, Newton simplified his treatment of massive bodies by considering them to
be mathematical points with no size or rotation. This allowed him to ignore factors such as friction, air
resistance, temperature, material properties, etc., and concentrate on phenomena that can be
described solely in terms of mass, length and time. Consequently, the three laws cannot be used to
describe precisely the behavior of large rigid or deformable objects; however, in many cases they
provide suitably accurate approximations.

Newton's laws pertain to the motion of massive bodies in an inertial reference frame, sometimes called
a Newtonian reference frame, although Newton himself never described such a reference frame. An
inertial reference frame can be described as a 3-dimensional coordinate system that is either stationary
or in uniform linear motion., i.e., it is not accelerating or rotating. He found that motion within such an
inertial reference frame could be described by three simple laws.
The First Law of Motion states, "A body at rest will remain at rest, and a body in motion will remain in
motion unless it is acted upon by an external force." This simply means that things cannot start, stop, or
change direction all by themselves. It takes some force acting on them from the outside to cause such a
change. This property of massive bodies to resist changes in their state of motion is sometimes
called inertia.

The Second Law of Motion describes what happens to a massive body when it is acted upon by an
external force. It states, "The force acting on an object is equal to the mass of that object times its
acceleration." This is written in mathematical form as F = ma, where F is force, m is mass, and a is
acceleration. The bold letters indicate that force and acceleration are vector quantities, which means
they have both magnitude and direction. The force can be a single force, or it can be the vector sum of
more than one force, which is the net force after all the forces are combined.

When a constant force acts on a massive body, it causes it to accelerate, i.e., to change its velocity, at a
constant rate. In the simplest case, a force applied to an object at rest causes it to accelerate in the
direction of the force. However, if the object is already in motion, or if this situation is viewed from a
moving reference frame, that body might appear to speed up, slow down, or change direction
depending on the direction of the force and the directions that the object and reference frame are
moving relative to each other.

The Third Law of Motion states, "For every action, there is an equal and opposite reaction." This law
describes what happens to a body when it exerts a force on another body. Forces always occur in pairs,
so when one body pushes against another, the second body pushes back just as hard. For example,
when you push a cart, the cart pushes back against you; when you pull on a rope, the rope pulls back
against you; when gravity pulls you down against the ground, the ground pushes up against your feet;
and when a rocket ignites its fuel behind it, the expanding exhaust gas pushes on the rocket causing it to
accelerate.

If one object is much, much more massive than the other, particularly in the case of the first object
being anchored to the Earth, virtually all of the acceleration is imparted to the second object, and the
acceleration of the first object can be safely ignored. For instance, if you were to throw a baseball to the
west, you would not have to consider that you actually caused the rotation of the Earth to speed up ever
so slightly while the ball was in the air. However, if you were standing on roller skates, and you threw a
bowling ball forward, you would start moving backward at a noticeable speed.

The three laws have been verified by countless experiments over the past three centuries, and they are
still being widely used to this day to describe the kinds of objects and speeds that we encounter in
everyday life. They form the foundation of what is now known as classical mechanics, which is the study
of massive objects that are larger than the very small scales addressed by quantum mechanics and that
are moving slower than the very high speeds addressed by relativistic mechanics.
Examples

1) A mass of 5 kg is suspended by a rope of length 2 m from the ceiling. A force of 45 N in the horizontal
direction is applied at the midpoint R of the rope, as shown. What is the angle the rope makes with the
vertical in equilibrium? (Take g = 10 ms-2). Neglect the mass of the rope.

Solution: We begin by drawing the free body diagram of the mass to find T2.
As the mass is in equilibrium, the sum of all the external forces on it should be zero.
Therefore, T2 = 50 N

Next, we draw the free body diagram of the point R.


Since, point P is also in equilibrium, the sum of all the forces at this point must be zero.
That is, T1Cosθ = 45 N
and T1Sinθ = 50 N
Therefore, tanθ = (50/45), θ = 48o

2) A mass of 3 kg rests on a horizontal plane. The plane is gradually inclined until at an angle θ = 20° with
the horizontal, the mass just begins to slide. What is the coefficient of static friction between the block
and the surface?

Solution: Again we begin by drawing a figure containing all the forces acting on the mass. Now, instead
of drawing another free body diagram, we should be able to see it in this figure itself.
An important thing to keep in mind here is that we have resolved the force of gravity into its
components and we must not consider mg during calculations if we are taking its components into
account.
Now, as θ increases, the self-adjusting frictional force fs increases until at θ = θmax, fs achieves its
maximum value, (fs)max = μsN.
Therefore, tanθmax = μs or θmax = tan–1μs
When θ becomes just a little more than θmax, there is a small net force on the block and it begins to slide.
Hence, for θmax = 20°,
μs = tan 20° = 0.36

3) A small block B is placed is placed on another block A of mass 7 kg and length 15 cm. Initially the block
B is near the right end of block A. A constant horizontal force of 10 N is applied to the block A. All the
surfaces are assumed frictionless. Find the time elapsed before the block B separates from A.

Solution: As there is no friction between A and B, when the block A moves, block B remains at rest in its
position.
Now, acceleration of block A = 10/7 = 1.4 ms-2
As the block starts from rest, initial velocity, u = 0
We know that s = ut + ½ at2
0.15 = 0 + 0.7t2
which gives t = 0.46 s

4) A ball of mass 5 kg and a block of mass 12 kg are attached by a lightweight cord that passes over a
frictionless pulley of negligible mass as shown in the figure. The block lies on a frictionless incline of
angle 30o. Find the magnitude of the acceleration of the two objects and the tension in the cord. Take g
= 10 ms-2.
Solution:

As the objects are connected by a cord (which we assume does not stretch), their accelerations have the
same magnitude. Let us draw the free body diagrams for the two objects.

Now, we know that the block never loses contact with the surface of the incline. Hence, in the direction
perpendicular to the surface the sum of forces on the block must be zero.

Therefore, normal force, n = 120Cos30 = 60√3 = 104 N


Also, 120Sin30 – T = ma = 12a
Therefore, a = (60-T)/12

Now, for the ball also, T – 50 = 5a (as both the objects have same acceleration)
Putting the value of a from the above equation, we get
T- 50 = 5(60-T)/12
i.e. 17T = 900
i.e. T = 52.94 N
Substituting the value of T in T – 50 = 5a, we get
a = 0.59 ms-2 ≈ 0.6 ms-2
Note that here we chose the x axis for the incline along the surface of the incline and the y axis
perpendicular to the surface of the incline. Such choices for the axes should be made which make the
problem easier to solve. It would have been a tedious problem if we had stick to the conventional choice
for the axes.

5) Two blocks of masses 5 kg and 7 kg are placed in contact with each other on a frictionless horizontal
surface. A constant horizontal force 20 N is applied to the block of mass 7 kg.
(a) Determine the magnitude of the acceleration of the two-block system.

(b) Determine the magnitude of the contact force between the two blocks.

Solution: We know that the blocks remain in contact from our everyday experience.
Now, given that the force of 20 N is the only force acting on the system in the horizontal direction and
we know that the blocks move together as one, we can assume the system to be a mass of 12 kg in
order to solve part (a).
Therefore, acceleration, a = 20/12 = 1.67 ms-2.
To solve part (b) we draw the free body diagrams of both the blocks.

Applying Newton’s second law to the block of mass 5 kg gives


∑Fx = P = 5a = 5 X 1.67 = 8.33 N
Now, to verify the value of P obtained apply Newton’s second law to the block of mass 7 kg,
∑Fx = 20 – P = 11.67 N
which gives a = 11.67/7 = 1.67 ms-2.
Hence, verified.

6) A block of mass 5 kg starts to slide down a frictionless plane having an inclination of 25.0° from rest at
the top. The length of the incline is 2.00 m, find (a) the acceleration of the block and (b) its speed when
it reaches the bottom of the incline. Take g = 10 ms-2.

Solution: We begin by drawing the free body diagram of the block.


(a) Applying Newton’s second law of motion along the surface of the incline, we get
50 sin25 = 5a
Therefore, a = 4.23 ms-2
(b) We know that v2 = u2 + 2as
Since u = 0 (given that the block starts from rest)
Therefore, v2 = 2 X 4.23 X 2
which gives v = 4.1 m/s
7) A 75.0 kg man stands on a platform scale in an elevator. Starting from rest, the elevator ascends,
attaining its maximum speed of 1.20 m/s in 1.00 s. It travels with this constant speed for the next 10.00
s. The elevator then undergoes a uniform acceleration in the negative y direction for 1.70 s and comes
to rest. What does the scale register
(a) before the elevator starts to move?
(b) during the first 1.00 s?
(c) while the elevator is traveling at constant speed?
(d) during the time it is slowing down? Take g = 10 ms-2.
Solution: The scale registers the force applied by it on the man i.e. the normal force. We proceed by
drawing the free body diagram for the person.

(a) Before the elevator starts to move, the man is in equilibrium. Hence the sum of all forces on him
must be zero.
Therefore, N = 750 N
Hence, the scale registers 750 N before the elevator starts to move.

(b) The man accelerates with the elevator at the same acceleration.
Applying Newton’s second law in the vertical direction, we get
∑Fy = 750 – N = 75a = 75 X 1.2
which gives N = 660 N
Hence, the spring scale registers 660 N for the first 1 s.

(c) While the elevator is travelling at constant speed, the acceleration of the man is zero. Therefore, the
forces applied on him must add up to zero.
which implies N = 750 N

(d) The maximum speed of the elevator, v = u + at


u = 0 (as the elevator starts from rest)
Therefore, v = 1.2 X 1 = 1.2 m/s
It then comes to rest in 1.7 s after decelerating uniformly.
i.e. 0 = 1.2 + a’t = 1.2 + a’ X 1.7
which gives a’ = -0.7 ms-2
Now, applying Newton’s second law in the vertical plane, we get,
∑Fy = 750 – N = 75a = 75 X (-0.7)
which gives N = 802.5 N

8) A block of mass 2.50 kg is accelerated across a rough surface by a rope passing over a pulley, as
shown in figure. The tension in the rope is 12.0 N, and the pulley is 10.0 cm above the top of the block.
The coefficient of kinetic friction is 0.300. (a) Determine the acceleration of the block when x = 0.400 m.
(b) Find the value of x at which the acceleration becomes zero. Take g = 10 ms-2.

Solution: To determine the acceleration of the block, we need to know all the forces acting on it.

At x = 0.4 m, cosθ = 40/41.23 = 0.97


Sinθ = 0.24
Since, T = 12 N, Tcosθ = 11.64 N
Tsinθ = 2.9 N
Applying Newton’s second law in the vertical direction, we get
∑Fy = 25 – N – 2.9 = 0 (as the block does not lose contact with the floor and hence its acceleration is zero
in this direction)
which gives N = 23.9 N

Therefore, Force of friction, fk = μk X 23.9 = .3 X 23.9 = 7.17 N


Now, applying Newton’s second law in the horizontal direction, we get
∑Fx = 11.64 – 7.17 = ma = 2.5a
which gives a = 1.8 ms-2.

(b) The acceleration becomes zero when Tcosθ equals frictional force
i.e. Tcosθ = μk X (25 – Tsinθ)
12cosθ = 0.3 X (25 – 12sinθ)
12cosθ + 3.6sinθ = 7.5
Sinαcosθ + cosαsinθ = 7.5/12.53
Where sinα = 12/12.53 and cosα = 3.6/12.53
So, sin(α + θ) = 0.6
i.e. α + θ = 36.7o
Also, sin(α + θ) = 10x2+102 = 0.6

Solving, we get x = 13.33 m

9) A block is given an initial velocity of 5.00 m/s up a frictionless 20.0° incline. How far up the incline
does the block slide before coming to rest? Take g = 10 ms-2.

Solution:
Initial velocity of the block, u = 5 m/s

The free body diagram of the block looks like


Therefore, the force acting on the block in its direction of motion is mg sinθ = mg sin20
Hence, a = -gsin20 taking positive x in the direction of motion
We know, that v2 = u2 + 2as
Therefore, 0 = 25 + 2 X (-g sin20) X s
which gives s = 3.7 m
Hence, the block comes to rest after travelling 3.7 m up the incline.
10) Two blocks of mass 5 kg and 9 kg are connected by a string of negligible mass that passes over a
frictionless pulley. The inclines are frictionless. Find (a) the magnitude of the acceleration of each block
and (b) the tension in the string. Take g = 10 ms-2.
Solution: Let the tension in the string be T and the acceleration of the blocks be ‘a’.
Both the blocks being connected by the string move with same acceleration. The heavier block slides
downwards ad the lighter block slides upwards the incline.
Applying Newton’s second law to both the blocks,
T – 50sin35 = 5a (for 5 kg mass)
90sin35 – T = 9a (for 9 kg mass)
Solving the two equations we get,
a = 10sin35 = 5.7 N
Therefore, T = 57 N

11) In figure blocks A and B have masses 40 kg and 20 kg respectively. (a) Determine the minimum
weight of block C to keep A from sliding if the coefficient of friction, μs between A and the table is 0.18.
Block C is suddenly lifted off A. (b) What is the acceleration of block A if μk between A and the table is

0.15. Take g = 10 ms-2.


Solution: (a) Let the minimum weight of block C to keep A from sliding be m.
If block A doesn’t slide, block B also remains in equilibrium, as they are connected by a string.
Therefore, tension in the string, T = 200 N

Now, considering the free body diagram of system A and C


Applying Newton’s second law in x direction, we have
fs = T = μs X N
i.e. 200 = 0.18 X (400 + mg)
m = 71.1 kg

(b) If block is lifted off suddenly, the normal force becomes


N = 400 N
And as the block stars sliding, the kinetic friction, fk= μk X 400 = 0.15 X 400 = 60 N
Therefore the acceleration of the block, a = (200-60)/40 = 3.5 ms-2.
12) Two masses 5 kg and 7 kg situated on a frictionless, horizontal surface are connected by a light
string. A force of 50 N is exerted on one of the masses to the right. Determine the acceleration of the
system and the tension T in the string.

Solution: The only external force acting on the system is 50 N in the x axis. As the blocks don’t lose
contact with the floor the normal force is balanced by their weight for both the blocks.
We could treat the system as one block of 12 kg as we can see that both the blocks have to have the
same acceleration.
Therefore, 50 = 12a
which gives a = 4.17 ms-2.

To find the tension T in the string, let us draw the free body diagrams for the two blocks

Applying Newton’s second law on block of mass 5 kg, we get

T = 5a = 5 X 4.17 = 20.8 N
To verify, apply Newton’s second law on block of mass 7 kg, which gives
50 – T = 7 X 4.17
T = 20.8 N
Hence verified.
13) A block of mass m1 = 5 kg on a frictionless horizontal table is connected to a block of mass m2 = 3 kg
by means of a very light pulley P1 and a light fixed pulley P2 as shown in figure. If a1 and a2 are the
accelerations of m1 and m2, respectively, (a) what is the relationship between these accelerations? Find
(b) the tensions in the strings and (c) the accelerations a1 and a2. Take g = 10 ms-
2
.

Solution: Let the tensions in the strings of P1 and P2 be T1 and T2 respectively.


For part (a), you need to visualize the problem carefully. Since the pulley P1 and mass m2 = 3 kg are
connected by a string, their accelerations have to be the same as they have to move together. Now if
the pulley goes forward a distance x, the mass m1 has to move forward by 2x in order for the string to
not break, x for both the sides (upper and lower) of the string, which implies that the mass m1 will have
to move with twice the acceleration as that of m2.
Hence, a1 = 2a2.
(b) Drawing the free body diagram of block m2, and applying Newton’s second law, we get

30 – T2 = 3 X a2 …(1)
Now, since the pulley P1 is massless, the sum of forces acting on it must be zero otherwise it will have
infinite acceleration which cannot happen without destroying the system. Hence,
2T1 = T2

Now for mass m1 we have


T1 = 5 X a1 = 10a2
or T2/2 = 10a2 …(2)
Solving equations (1) and (2);
a2 = 1.3 ms-2
Therefore, a1 = 2.6 ms-2
Using this value of a2, we get
T1 = 13 N
and T2 = 26 N

14) A block A of mass 0.5 kg can slide on a frictionless incline of angle 30o and length 0.8 m kept inside
an elevator going up with uniform velocity 2m/s. Find the time taken by the block to slide down the
length of the incline if it is released from the top of the incline. Take g = 10 ms-2.

S
Solution: The driving force on the block which causes it to move down the plane is
F = mgsin30 = 2.5 N
So, acceleration = 2.5/0.5 = 5 ms-2
Initial velocity of block u = 0
s = 0.8 m
Now, we that s = ut + ½ at2
0.8 = 0 + 2.5t2
which gives t = 0.57 s

15) The elevator shown is descending with a constant acceleration of 1.5 ms-2. The mass of the block A is
1 kg. What force is exerted by block A on block B? Take g = 10 ms-2.
Solution: The acceleration of the two blocks is the same as that of the elevator as they do not lose

contact with the floor of the elevator. The free body diagram of the block A is
Applying Newton’s second law of motion, we have,
10 – N = 1.5
which gives N = 6.67 N
Now, from Newton’s third law of motion, the force exerted by block A on block B = N = 6.67 N
Kinetics and kinematics
Kinetics and kinematics is a pair of terms which is often confusing. Whenever we come across the terms
like these get very puzzled as to which one is right or even if both are right which one is more
appropriate to use within the given context. Some brilliant minds think it as a dilemma between
American and British English without even googling it!!! Well, let me just tell you – this is not the case.
Let’s try to find out the actual difference between kinetics and kinematics and fix this mix-up once and
for all.

Mechanics is the one umbrella term which includes two branches namely, statics and dynamics and
dynamics is further divided into two sub-branches called kinetics and kinematics.

I hope you’ve got the idea that kinetics and kinematics are not synonymous. Now, let’s dig a little
deeper and see in brief what these highlighted terms mean.

Mechanics is a branch of physics concerned with the study of an object under the influence of an
external quantity. This external quantity can be a force, a torque (a twisting force) or momentum

Statics is the division of mechanics which deals with a body in equilibrium and when I say equilibrium, I
mean the body is either at rest or in uniform motion i.e. moving with a constant velocity.

Dynamics deals with the objects under accelerated motion due to the effect of a force. The
key difference between statics and dynamics is that in statics there is no acceleration. Dynamics of a
body considers its motion and the underlying reason behind motion e.g. the force.

Kinematics is concerned with the study of the motion of a point, a body or a system of bodies without
bearing in mind the cause of motion i.e., force. (kinematics does not take into account body’s mass also
because that would mean taking force into account indirectly, REMEMBER F=m∗a ). Hence, we can say
that while dealing with the kinematics of a body we are interested in pure motion only. Kinematics is not
curious about “how the change?” but “how much change?”

In order to describe the motion of a body we put into service following well-known equations,
therefore, these equations are better known as kinematic equations and not kinetic equations.

Kinematic equations
v = u + at [velocity-time relation]

s = ut + 1/2 at^2 [position-time relation]

v^2 = u^2+ 2as [position-velocity relation]

here –

symbol ‘v’ stands for final velocity

the symbol ‘u’ stands for initial velocity

the symbol ‘a’ stands for acceleration

symbol ‘s’ stands for displacement

symbol ‘t’ stands for time as a reference

Given to its mathematical niche kinematics is also considered as another branch of mathematics.

Kinematics is often referred to as geometry of motion for it focuses on the geometrical aspects of the
motion.

Kinetics is a vast field as it used not only in physics but also in chemistry and biology. In pure physics
terms, it talks about the motion of a body taking into account the cause of motion. In kinetics the cause
of motion (force) is studied extensively, this is the key difference between kinematics and
kinetics. Newton’s laws of motion are employed extensively in kinetics. In terms of chemistry, it gives
the rate of a chemical reaction. Kinetics is used in biology in terms of enzyme kinetics as to how an
enzyme affects the rate of a biochemical reaction.

You may find in some text that the term kinetics is suppressed by the term dynamics and this has been
done to eliminate the ambiguity between the two terms. Now, as you know the difference between
kinetics and kinematics clearly, this ambiguity should not exist.
Examples

1. An airplane accelerates down a runway at 3.20 m/s2 for 32.8 s until is finally lifts off the ground.
Determine the distance traveled before takeoff.

Given:

a = +3.2 m/s2 t = 32.8 s vi = 0 m/s Find:d = ??

d = vi*t + 0.5*a*t2
d = (0 m/s)*(32.8 s)+ 0.5*(3.20 m/s2)*(32.8 s)2
d = 1720 m

2. A car starts from rest and accelerates uniformly over a time of 5.21 seconds for a distance of
110 m. Determine the acceleration of the car.

Given: Find:a = ??
d = 110 m t = 5.21 s vi = 0 m/s
d = vi*t + 0.5*a*t2
110 m = (0 m/s)*(5.21 s)+ 0.5*(a)*(5.21 s)2
110 m = (13.57 s2)*a
a = (110 m)/(13.57 s2)
a = 8.10 m/ s2

3. Upton Chuck is riding the Giant Drop at Great America. If Upton free falls for 2.60 seconds, what
will be his final velocity and how far will he fall?

Given: Find: d = ?? vf = ??
a = -9.8 m t = 2.6 s vi = 0 m/s
d = vi*t + 0.5*a*t2
d = (0 m/s)*(2.60 s)+ 0.5*(-9.8 m/s2)*(2.60 s)2
d = -33.1 m (- indicates direction)
vf = vi + a*t
vf = 0 + (-9.8 m/s2)*(2.60 s)
vf = -25.5 m/s (- indicates direction)
4. A race car accelerates uniformly from 18.5 m/s to 46.1 m/s in 2.47 seconds. Determine the
acceleration of the car and the distance traveled.

Given: Find: d = ??
vi = 18.5 m/s vf = 46.1 m/s t = 2.47 s a = ??
a = (Delta v)/t
a = (46.1 m/s - 18.5 m/s)/(2.47 s)
a = 11.2 m/s2
d = vi*t + 0.5*a*t2
d = (18.5 m/s)*(2.47 s)+ 0.5*(11.2 m/s2)*(2.47 s)2
d = 45.7 m + 34.1 m
d = 79.8 m
(Note: the d can also be calculated using the equation vf2 = vi2 + 2*a*d)

5. A feather is dropped on the moon from a height of 1.40 meters. The acceleration of gravity on
the moon is 1.67 m/s2. Determine the time for the feather to fall to the surface of the moon.

Given: Find: t = ??
vi = 0 m/s d = -1.40 m a = -1.67 m/s2
d = vi*t + 0.5*a*t2
-1.40 m = (0 m/s)*(t)+ 0.5*(-1.67 m/s2)*(t)2
-1.40 m = 0+ (-0.835 m/s2)*(t)2
(-1.40 m)/(-0.835 m/s2) = t2
1.68 s2 = t2
t = 1.29 s
6. Rocket-powered sleds are used to test the human response to acceleration. If a rocket-powered
sled is accelerated to a speed of 444 m/s in 1.83 seconds, then what is the acceleration and
what is the distance that the sled travels?

Given: Find: a = ??d = ??


vi = 0 m/s vf = 444 m/s t = 1.83 s
a = (Delta v)/t
a = (444 m/s - 0 m/s)/(1.83 s)
a = 243 m/s2
d = vi*t + 0.5*a*t2
d = (0 m/s)*(1.83 s)+ 0.5*(243 m/s2)*(1.83 s)2
d = 0 m + 406 m
d = 406 m
(Note: the d can also be calculated using the equation vf2 = vi2 + 2*a*d)

7. A bike accelerates uniformly from rest to a speed of 7.10 m/s over a distance of 35.4 m.
Determine the acceleration of the bike.

Given: Find: a = ??
vi = 0 m/s vf = 7.10 m/s d = 35.4 m
vf2 = vi2 + 2*a*d
(7.10 m/s)2 = (0 m/s)2 + 2*(a)*(35.4 m)
50.4 m2/s2 = (0 m/s)2 + (70.8 m)*a
(50.4 m2/s2)/(70.8 m) = a
a = 0.712 m/s2
8. An engineer is designing the runway for an airport. Of the planes that will use the airport, the
lowest acceleration rate is likely to be 3 m/s2. The takeoff speed for this plane will be 65 m/s.
Assuming this minimum acceleration, what is the minimum allowed length for the runway?

Given: Find: d = ??
vi = 0 m/s vf = 65 m/s a = 3 m/s2
vf2 = vi2 + 2*a*d
(65 m/s)2 = (0 m/s)2 + 2*(3 m/s2)*d
4225 m2/s2 = (0 m/s)2 + (6 m/s2)*d
(4225 m2/s2)/(6 m/s2) = d
d = 704 m

9. A car traveling at 22.4 m/s skids to a stop in 2.55 s. Determine the skidding distance of the car
(assume uniform acceleration).

Given: Find: d = ??
vi = 22.4 m/s vf = 0 m/s t = 2.55 s
d = (vi + vf)/2 *t
d = (22.4 m/s + 0 m/s)/2 *2.55 s
d = (11.2 m/s)*2.55 s
d = 28.6
10. A kangaroo is capable of jumping to a height of 2.62 m. Determine the takeoff speed of the
kangaroo.

Given: Find: vi = ??
a = -9.8 m/s2 vf = 0 m/s d = 2.62 m
vf2 = vi2 + 2*a*d
(0 m/s)2 = vi2 + 2*(-9.8 m/s2)*(2.62 m)
0 m2/s2 = vi2 - 51.35 m2/s2
51.35 m2/s2 = vi2
vi = 7.17 m/s

11. If Michael Jordan has a vertical leap of 1.29 m, then what is his takeoff speed and his hang time
(total time to move upwards to the peak and then return to the ground)?

Given: Find: vi = ??t = ??


a = -9.8 m/s2 vf = 0 m/s d = 1.29 m
vf2 = vi2 + 2*a*d
(0 m/s)2 = vi2 + 2*(-9.8 m/s2)*(1.29 m)
0 m2/s2 = vi2 - 25.28 m2/s2
25.28 m2/s2 = vi2
vi = 5.03 m/s
To find hang time, find the time to the peak and then double it.
vf = vi + a*t
0 m/s = 5.03 m/s + (-9.8 m/s2)*tu
-5.03 m/s = (-9.8 m/s2)*tup
(-5.03 m/s)/(-9.8 m/s2) = tup
tup = 0.513 s
hang time = 1.03 s
12. A bullet leaves a rifle with a muzzle velocity of 521 m/s. While accelerating through the barrel of
the rifle, the bullet moves a distance of 0.840 m. Determine the acceleration of the bullet
(assume a uniform acceleration).

Given: Find: a = ??
vi = 0 m/s vf = 521 m/s d = 0.840 m
vf2 = vi2 + 2*a*d
(521 m/s)2 = (0 m/s)2 + 2*(a)*(0.840 m)
271441 m2/s2 = (0 m/s)2 + (1.68 m)*a
(271441 m2/s2)/(1.68 m) = a
a = 1.62*105 m /s2

13. A baseball is popped straight up into the air and has a hang-time of 6.25 s. Determine the height
to which the ball rises before it reaches its peak. (Hint: the time to rise to the peak is one-half
the total hang-time.)

Given: Find: d = ??
a = -9.8 m/s2 vf = 0 m/s t = 3.13 s
1. (NOTE: the time required to move to the peak of the trajectory is one-half the total hang time -
3.125 s.)

First use: vf = vi + a*t


0 m/s = vi + (-9.8 m/s2)*(3.13 s)
0 m/s = vi - 30.7 m/s
vi = 30.7 m/s (30.674 m/s)
Now use: vf2 = vi2 + 2*a*d
(0 m/s)2 = (30.7 m/s)2 + 2*(-9.8 m/s2)*(d)
0 m2/s2 = (940 m2/s2) + (-19.6 m/s2)*d
-940 m2/s2 = (-19.6 m/s2)*d
(-940 m2/s2)/(-19.6 m/s2) = d
d = 48.0 m
14. The observation deck of tall skyscraper 370 m above the street. Determine the time required for
a penny to free fall from the deck to the street below.

Given: Find: t = ??
vi = 0 m/s d = -370 m a = -9.8 m/s2
d = vi*t + 0.5*a*t2
-370 m = (0 m/s)*(t)+ 0.5*(-9.8 m/s2)*(t)2
-370 m = 0+ (-4.9 m/s2)*(t)2
(-370 m)/(-4.9 m/s2) = t2
75.5 s2 = t2
t = 8.69 s

15. A bullet is moving at a speed of 367 m/s when it embeds into a lump of moist clay. The bullet
penetrates for a distance of 0.0621 m. Determine the acceleration of the bullet while moving
into the clay. (Assume a uniform acceleration.)

Given: Find: a = ??
vi = 367 m/s vf = 0 m/s d = 0.0621 m
vf2 = vi2 + 2*a*
(0 m/s)2 = (367 m/s)2 + 2*(a)*(0.0621 m)
0 m2/s2 = (134689 m2/s2) + (0.1242 m)*a
-134689 m2/s2 = (0.1242 m)*a
(-134689 m2/s2)/(0.1242 m) = a
a = -1.08*106 m /s2
(The - sign indicates that the bullet slowed down.)
Motion Of A Particle In One Dimension

Uniform motion
According to Newton’s first law (also known as the principle of inertia), a body with no net force acting
on it will either remain at rest or continue to move with uniform speed in a straight line, according to its
initial condition of motion. In fact, in classical Newtonian mechanics, there is no important distinction
between rest and uniform motion in a straight line; they may be regarded as the same state of motion
seen by different observers, one moving at the same velocity as the particle, the other moving at
constant velocity with respect to the particle.
Although the principle of inertia is the starting point and the fundamental assumption of classical
mechanics, it is less than intuitively obvious to the untrained eye. In Aristotelian mechanics, and in
ordinary experience, objects that are not being pushed tend to come to rest. The law of inertia was
deduced by Galileo from his experiments with balls rolling down inclined planes, described above.
For Galileo, the principle of inertia was fundamental to his central scientific task: he had to explain how
it is possible that if Earth is really spinning on its axis and orbiting the Sun we do not sense that motion.
The principle of inertia helps to provide the answer: Since we are in motion together with Earth, and our
natural tendency is to retain that motion, Earth appears to us to be at rest. Thus, the principle of inertia,
far from being a statement of the obvious, was once a central issue of scientific contention. By the time
Newton had sorted out all the details, it was possible to account accurately for the small deviations from
this picture caused by the fact that the motion of Earth’s surface is not uniform motion in a straight line
(the effects of rotational motion are discussed below). In the Newtonian formulation, the common
observation that bodies that are not pushed tend to come to rest is attributed to the fact that they have
unbalanced forces acting on them, such as friction and air resistance.
As has already been stated, a body in motion may be said to have momentum equal to the product of
its mass and its velocity. It also has a kind of energy that is due entirely to its motion, called kinetic
energy. The kinetic energy of a body of mass m in motion with velocity v is given by

Falling bodies and uniformly accelerated motion

During the 14th century, the French scholar Nicole Oresme studied the mathematical properties
of uniformly accelerated motion. He had little interest in whether that kind of motion could be observed
in the realm of actual human existence, but he did discover that, if a particle is uniformly accelerated, its
speed increases in direct proportion to time, and the distance it traverses is proportional to the square
of the time spent accelerating. Two centuries later, Galileo repeated these same mathematical
discoveries (perhaps independently) and, just as important, determined that this kind of motion is
actually executed by balls rolling down inclined planes. As the incline of the plane increases,
the acceleration increases, but the motion continues to be uniformly accelerated. From this observation,
Galileo deduced that a body falling freely in the vertical direction would also have uniform acceleration.
Even more remarkably, he demonstrated that, in the absence of air resistance, all bodies would fall with
the same constant acceleration regardless of their mass. If the constant acceleration of any body
dropped near the surface of Earth is expressed as g, the behaviour of a body dropped from rest at
height z0 and time t = 0 may be summarized by the following equations:
where z is the height of the body above the surface, v is its speed, and a is its acceleration. These
equations of motion hold true until the body actually strikes the surface. The value of g is approximately
9.8 metres per second squared (m/s2).
A body of mass m at a height z0 above the surface may be said to possess a kind of energy purely by
virtue of its position. This kind of energy (energy of position) is called potential energy. The gravitational
potential energy is given by

Technically, it is more correct to say that this potential energy is a property of the Earth-body system
rather than a property of the body itself, but this pedantic distinction can be ignored.
As the body falls to height z less than z0, its potential energy U converts to kinetic energy K = 1/2mv2.
Thus, the speed v of the body at any height z is given by solving the equation

Equation (8) is an expression of the law of conservation of energy. It says that the sum of kinetic
energy, 1/2mv2, and potential energy, mgz, at any point during the fall, is equal to the total initial
energy, mgz0, before the fall began. Exactly the same dependence of speed on height could be deduced
from the kinematic equations (4), (5), and (6) above.

In order to reach the initial height z0, the body had to be given its initial potential energy by some
external agency, such as a person lifting it. The process by which a body or a system obtains mechanical
energy from outside of itself is called work. The increase of the energy of the body is equal to the work
done on it. Work is equal to force times distance.
The force exerted by Earth’s gravity on a body of mass m may be deduced from the observation that the
body, if released, will fall with acceleration g. Since force is equal to mass times acceleration, the force
of gravity is given by F = mg. To lift the body to height z0, an equal and opposite (i.e., upward) force must
be exerted through a distance z0. Thus, the work done is

which is equal to the potential energy that results.

If work is done by applying a force to a body that is not being acted upon by an opposing force, the body
is accelerated. In this case, the work endows the body with kinetic energy rather than potential energy.
The energy that the body gains is equal to the work done on it in either case. It should be noted that
work, potential energy, and kinetic energy, all being aspects of the same quantity, must all have the
dimensions ml2/t2.

Simple harmonic oscillations


Consider a mass m held in an equilibrium position by springs, as shown in Figure 2A. The mass may be
perturbed by displacing it to the right or left. If x is the displacement of the mass from equilibrium
(Figure 2B), the springs exert a force F proportional to x, such that

Figure 2: (A) A mass m held in equilibrium by springs. (B) A mass m displaced a distance x.Encyclopædia
Britannica, Inc.

where k is a constant that depends on the stiffness of the springs. Equation (10) is called Hooke’s law,
and the force is called the spring force. If x is positive (displacement to the right), the resulting force is
negative (to the left), and vice versa. In other words, the spring force always acts so as to restore mass
back toward its equilibrium position. Moreover, the force will produce an acceleration along
the x direction given by a = d2x/dt2. Thus, Newton’s second law, F = ma, is applied to this case by
substituting −kx for F and d2x/dt2 for a, giving −kx = m(d2x/dt2). Transposing and dividing by m yields the
equation

Equation (11) gives the derivative—in this case the second derivative—of a quantity x in terms of the
quantity itself. Such an equation is called a differential equation, meaning an equation containing
derivatives. Much of the ordinary, day-to-day work of theoretical physics consists of solving differential
equations. The question is, given equation (11), how does x depend on time?

The answer is suggested by experience. If the mass is displaced and released, it will oscillate back and
forth about its equilibrium position. That is, x should be an oscillating function of t, such as a
sine wave or a cosine wave. For example, x might obey a behaviour such as

Equation (12) describes the behaviour sketched graphically in Figure 3. The mass is initially displaced a
distance x = A and released at time t = 0. As time goes on, the mass oscillates from A to −A and back
to A again in the time it takes ωt to advance by 2π. This time is called T, the period of oscillation, so that
ωT = 2π, or T = 2π/ω. The reciprocal of the period, or the frequency f, in oscillations per second, is given
by f = 1/T = ω/2π. The quantity ω is called the angular frequency and is expressed in radians per second.


Figure 3: The function x = A cos ωt.Encyclopædia Britannica, Inc.
The choice of equation (12) as a possible kind of behaviour satisfying the differential equation (11) can
be tested by substituting it into equation (11). The first derivative of x with respect to t is

Differentiating a second time gives

Equation (14) is the same as equation (11) if

Thus, subject to this condition, equation (12) is a correct solution to the differential equation. There are
other possible correct guesses (e.g., x = A sin ωt) that differ from this one only in whether the mass is at
rest or in motion at the instant t = 0.
The mass, as has been shown, oscillates from A to −A and back again. The speed, given by dx/dt,
equation (13), is zero at A and −A, but has its maximum magnitude, equal to ωA, when x is equal to zero.
Physically, after the mass is displaced from equilibrium a distance A to the right, the restoring
force F pushes the mass back toward its equilibrium position, causing it to accelerate to the left. When it
reaches equilibrium, there is no force acting on it at that instant, but it is moving at speed ωA, and
its inertia takes it past the equilibrium position. Before it is stopped it reaches position −A, and by this
time there is a force acting on it again, pushing it back toward equilibrium.
The whole process, known as simple harmonic motion, repeats itself endlessly with a frequency given by
equation (15). Equation (15) means that the stiffer the springs (i.e., the larger k), the higher the
frequency (the faster the oscillations). Making the mass greater has exactly the opposite effect, slowing
things down.

One of the most important features of harmonic motion is the fact that the frequency of the motion, ω
(or f), depends only on the mass and the stiffness of the spring. It does not depend on
the amplitude A of the motion. If the amplitude is increased, the mass moves faster, but the time
required for a complete round trip remains the same. This fact has profound consequences, governing
the nature of music and the principle of accurate timekeeping.
The potential energy of a harmonic oscillator, equal to the work an outside agent must do to push the
mass from zero to x, is U = 1/2kx2. Thus, the total initial energy in the situation described above is 1/2kA2;
and since the kinetic energy is always 1/2mv2, when the mass is at any point x in the oscillation,

Equation (16) plays exactly the role for harmonic oscillators that equation (8) does for falling bodies.

It is quite generally true that harmonic oscillations result from disturbing any body or structure from a
state of stable mechanical equilibrium. To understand this point, a brief discussion of stability is useful.
Consider a bowl with a marble resting inside, then consider a second, inverted bowl with a marble
balanced on top. In both cases, the net force on the marble is zero. The marbles are thus in
mechanical equilibrium. However, a small disturbance in the position of the marble balanced on top of
the inverted bowl will cause it to roll away and not return. In such a case, the equilibrium is said to be
unstable. Conversely, if the marble inside the first bowl is disturbed, gravity acts to push it back toward
the bottom of the bowl. The marble inside the bowl (like the mass held by springs in Figure 2A) is an
example of a body in stable equilibrium. If it is disturbed slightly, it executes harmonic oscillations
around the bottom of the bowl rather than rolling away.
This argument may be generalized by a simple mathematical argument. Consider a body or structure in
mechanical equilibrium, which, when disturbed by a small amount x, finds a force acting on it that is a
function of x, F(x). For small x, such a function may be written generally as a power series in x; i.e.,

where F(0) is the value of F(x) when x = (0), and a and b are constants, independent of x, determined by
the nature of the system. The statement that the body is in mechanical equilibrium means that F(0) = 0,
so that no force is acting on the body when it is undisturbed. Since x is small, x2 is much smaller; thus,
the term bx2 and all higher powers may be disregarded. This leaves F(x) = ax. Now, if a is positive, a
disturbance produces a force in the same direction as the disturbance. This was the case when the
marble was balanced on top of the inverted bowl. It describes unstable equilibrium. For the system to
be stable, a must be negative. Thus, if a = −k, where k is some positive constant, equation (17)
becomes F(x) = −kx, which is simply Hooke’s law, equation (10). As has been described above, any
system obeying Hooke’s law is a harmonic oscillator.

The generality of this argument accounts for the fact that harmonic oscillators are abundantly observed
in common experience. For example, any rigid structure will oscillate at many different
harmonic frequencies corresponding to different possible distortions of its equilibrium shape. In
addition, music may be produced either by disturbing the equilibrium of a stretched wire or fibre (as in
the piano and violin), a stretched membrane (e.g., drums), or a rigid bar (the triangle and the xylophone)
or by disturbing the density of an enclosed column of air (as in the trumpet and organ). While
a fluid such as air is not rigid, its density is an example of a stable system that obeys Hooke’s law and
may therefore be set into harmonic oscillations.
All music would be quite different from what it is were it not for the general property of harmonic
oscillators that the frequency is independent of the amplitude. Thus, instruments yield the same note
(frequency) regardless of how loudly they are played (amplitude), and, equally important, the same note
persists as the vibrations die away. This same property of harmonic oscillators is the underlying principle
of all accurate timekeeping.

The first precise timekeeping mechanism, whose principles of motion were discovered by Galileo, was
the simple pendulum (see below). The accuracy of modern timekeeping has been improved dramatically
by the introduction of tiny quartz crystals, whose harmonic oscillations generate electrical signals that
may be incorporated into miniaturized circuits in clocks and wristwatches. All harmonic oscillators are
natural timekeeping devices because they oscillate at intrinsic natural frequencies independent of
amplitude. A given number of complete cycles always corresponds to the same elapsed time. Quartz
crystal oscillators make more accurate clocks than pendulums do principally because they oscillate many
more times per second.
Examples:

1) The graph in Figure 2 below shows a particle’s velocity moving along a coordinate line.
At t=0, the position is x=0.
 Sketch the acceleration a vs. time t graph corresponding to this velocity vs. time graph;
 Sketch the graph of position x vs. time t corresponding to the velocity vs. time graph;
 Determine the average speed of the particle between t=0 and t=60sec.

Solution.
Charles’s Law (Problem-Solving)

1. A 600 mL sample of nitrogen is heated from 27 °C to 77 °C at constant pressure. What is the


final volume?
Solution:
The first step to solving gas law problems should be converting all temperatures to absolute
temperatures. In other words, if the temperature is given in Celsius or Fahrenheit, convert it
to Kelvin. This is the most common place mistakes are made in this type of homework
problem.
T K = 273 + °C
Ti = initial temperature = 27 °C
Ti K = 273 + 27
Ti K = 300 K
Tf = final temperature = 77 °C
Tf K = 273 + 77
Tf K = 350 K
The next step is to use Charles' law to find the final volume. Charles' law is expressed as:
Vi/Ti = Vf/Tf
where
Vi and Ti is the initial volume and temperature
Vf and Tf is the final volume and temperature
Solve the equation for Vf:
Vf = ViTf/Ti
Enter the known values and solve for Vf.
Vf = (600 mL)(350 K)/(300 K)
Vf = 700 mL
Answer:
The final volume after heating will be 700 mL.
2. A container holds 50.0 mL of nitrogen at 25° C and a pressure of 736 mm Hg. What will be
its volume if the temperature increases by 35° C?
P1 = 736 mm Hg
P2 = 736 mm Hg
V1 = 50.0 mL
V2 = ? T1 = 25° C + 273 = 298 K
T2 = 25° C + 35° C + 273 = 333 K
V1/ T1 = V2/T2 V2 = V1 x T2/T1
V2 = 50.0 mL x 333 K/298
Answer:
K = 55.9 mL N2

3. A sample of helium has a volume of 521 dm3 at a pressure of 75 cm Hg and a temperature


of 18° C. When the temperature is increased to 23° C, what is the volume of the helium?
P1 = 75 cm Hg
P2 = 75 cm Hg
V1 = 521 dm3
V2 = ? T1 = 18° C + 273 = 291 K
T2 = 23° C + 273 = 296 K
V1/ T1 = V2/T2
V2 = V1 x T2/T1
V2 = 521 dm3 x 296 K/291
Answer:
K = 530. dm3 He
4. 568 cm3 of chlorine at 25° C will occupy what volume at -25° C while the pressure remains
constant?
V1 = 568 cm3
V2 = ?
T1 = 25° C + 273 = 298 K
T2 = -25° C + 273 = 248 K
V1/ T1 = V2/T2
V2 = V1 x T2/T1
V2 = 568 cm3 x 248 K/298
Answer:
K = 473 cm3 Cl2

5. A sample of oxygen occupies a volume of 160 dm3 at 91° C. What will be volume of oxygen
when the temperature drops to 0.00° C?

V1 = 160 dm3
V2 = ?
T1 = 91° C + 273 = 364 K
T2 = 0.00° C + 273 = 273 K
V1/ T1 = V2/T2
V2 = V1 x T2/T1
V2 = 160 dm3 x 273 K/364
Answer:
K = 120 dm3 O2
6. A balloon is filled to a volume of 2.20L2.20L at a temperature of 22oC22oC. The balloon is
then heated to a temperature of 71oC71oC. Find the new volume of the balloon.

Find: V2 = ? L

Answer:

1.82 L
7. A sample of a gas has an initial volume of 34.8 L and an initial temperature of −67°C. What
must be the temperature of the gas for its volume to be 25.0 L?

Given:
T1 = -27oC and V1 = 34.8 L
V2 = 25.0 L
Find: T2 = ? K

K = -27oC + 273

Answer:
235 K, or −38°C
8. 5 moles of nitrogen gas is in a 100 liter fixed cylinder at 300 Kelvin. What is the pressure of
the gas?
It's often a good idea to rearrange equations to solve for the variable being asked about. In this
case, we need to get the equation in the form of P = . . . We'll do that after we organize the
values we'll need.

 P=?
 V = 100 L
 n = 5 moles
 R = 0.08205 L⋅atm / mole⋅K
 T = 300 K

Alright, so let's begin with formula PV = nRT and change that appropriately so it now is P =
nRT/V. Now we can plug in the values to determine the pressure. We get:

9. A 50 L sealed cylinder contains 100 moles of helium gas. Its pressure at 273 K is 44.80 atm.
What is the change in pressure when the gas is heated to 500 K?
This problem is a ''before and after'' scenario. We need the change in pressure (ΔP), which is
the initial pressure subtracted from the final pressure (Pf - Pi). As before, we'll organize the
values we'll need.

 ΔP=?
 Pf = ?
 Pi = 44.80 atm
 V = 50 L constant
 n = 100 moles constant
 R = 0.08205 L⋅atm / mole⋅K
 Ti = 273 K
 Tf = 500 K
Notice that we don't have initial and final subscripts for the molar amount of gas and volume.
This is because the number of moles of gas doesn't change. The volume of the gas can't change
either because it's a sealed cylinder. The only things that can change are the temperature and
pressure. Plugging in the numbers is next and we get:

10. I'm going to isolate T2 on one side of the equals sign:

Since the volume never changes, we can eliminate it from the equation:
P1 P2
––––– = –––––
n 1T 1 n 2T 2

Now, cross-multiply:

P1n2T2 = P2n1T1

Isolate T2:

P2n1T1
T2 = –––––
P 1n 2

Another way to write it is this:

T2 = P2n1T1 / P1n2
Avogadro's Law (Problem-Solving)

1. A 6.0 L sample at 25°C and 2.00 atm of pressure contains 0.5 mole of a gas. If an additional
0.25 mole of gas at the same pressure and temperature are added, what is the final total
volume of the gas?

Solution

First, express Avogadro's law by its formula:

Vi/ni = Vf/nf
where
Vi = initial volume
ni = initial number of moles
Vf = final volume
nf = final number of moles

For this example, Vi = 6.0 L and ni = 0.5 mole. When 0.25 mole is added:

nf = ni + 0.25 mole
nf = 0.5 mole = 0.25 mole
nf = 0.75 mole

The only variable remaining is the final volume.

Vi/ni = Vf/nf

Solve for Vf

Vf = Vinf/ni
Vf = (6.0 L x 0.75 mole)/0.5 mole
Vf = 4.5 L/0.5 Vf = 9 L

Check to see if the answer makes sense. You would expect the volume to increase if more
gas is added. Is the final volume greater than the initial volume? Yes. Doing this check is
useful because it is easy to put the initial number of moles in the numerator and the final
number of moles in the denominator. If this had happened, the final volume answer would
have been smaller than the initial volume.
2.

3. 5.00 L of a gas is known to contain 0.965 mol. If the amount of gas is increased to 1.80 mol,
what new volume will result (at an unchanged temperature and pressure)?

Solution:
I'll use V1n2 = V2n1
(5.00 L) (1.80 mol) = (x) (0.965 mol)
x = 9.33 L (to three sig figs)

4. A cylinder with a movable piston contains 2.00 g of helium, He, at room temperature. More
helium was added to the cylinder and the volume was adjusted so that the gas pressure
remained the same. How many grams of helium were added to the cylinder if the volume
was changed from 2.00 L to 2.70 L? (The temperature was held constant.)

Solution:
1) Convert grams of He to moles:
2.00 g / 4.00 g/mol = 0.500 mol
2) Use Avogadro's Law:
V1 / n1 = V2 / n2
2.00 L / 0.500 mol = 2.70 L / x
x = 0.675 mol
3) Compute grams of He added:
0.675 mol - 0.500 mol = 0.175 mol
0.175 mol x 4.00 g/mol = 0.7 grams of He added

5. : A flexible container at an initial volume of 5.120 L contains 8.500 mol of gas. More gas is
then added to the container until it reaches a final volume of 18.10 L. Assuming the
pressure and temperature of the gas remain constant, calculate the number of moles of gas
added to the container.

Solution:
V1 / n1 = V2 / n2
5.120 L 18.10 L
–––––––– = ––––––
8.500 mol x

x = 30.05 mol <--- total moles, not the moles added


30.05 - 8.500 = 21.55 mol (to four sig figs)
Notice the specification in the problem to determine moles of gas added. The Avogadro Law
calculation gives you the total moles required for that volume, NOT the moles of gas added.
That's why the subtraction is there.
6. If 0.00810 mol neon gas at a particular temperature and pressure occupies a volume of 214
mL, what volume would 0.00684 mol neon gas occupy under the same conditions?

Solution:
1) Notice that the same conditions are the temperature and pressure. Holding those two
constant means the volume and the number of moles will vary. The gas law that describes
the volume-mole relationship is Avogadro's Law:

V1 V2
––––– = ––––––
n1 n2

2) Substituting values gives:

214 mL V2
––––––––– = ––––––––––
0.00810 mol 0.00684 mol

3) Cross-multiply and divide for the answer:


V2 = 181 mL (to three sig figs)
When I did the actual calculation for this answer, I used 684 and 810 when entering
values into the calculator.
4) You may find this answer interesting:
Dividing PV1 = n1RT by PV2 = n2RT, we get
V1/V2 = n1/n2
V2 = V1n2/n1
V2 = (214 mL * 0.00684 mol) / 0.00810 mol
V2 = 181 mL
In case you don't know, PV = nRT is called the Ideal Gas Law. You'll see it a bit later in
your Gas Laws unit.
7. If 1.0mol of helium gas (He) at standard temperature and pressure (STP) has a
volume of 22.4L, how many moles of carbon tetrachloride gas (CCl4) will be present
in a container with a volume of 22.4L?

Explanation:
Avogadro's Law states that two gases at the same temperature and volume will have an
equal number of molecules, and therefore the same number of moles.
V1n1=V2n2
It does not matter that helium has one atom per molecule while carbon tetrachloride has
five atoms per molecule. Note that the given volume per mole of gas at STP is standard, and
true for any gas. At STP, one mole of gas will always have a volume of 22.5L, regardless of
its identity.

answer:

1.0mol

8. One mole of helium gas fills up an empty balloon to a volume of 1.5 litres. What would be
the volume of the balloon if an additional 2.5 moles of helium gas is added? (Assume that
the temperature and the pressure are kept constant)

Given,
The initial amount of helium (n1) = 1 mol
The initial volume of the balloon (V1) = 1.5 L
The final amount of helium (n2) = 1 mol + 2.5 mol = 3.5 mol
As per Avogadro’s law, V1/n1 = V2/n2
Therefore, the final volume of the balloon (V2) = (V1n2)/n1 = (1.5L*3.5mol)/1mol = 5.25 L
The balloon would occupy a volume of 5.25 liter when it contains 3.5 moles of helium.
9. A tyre containing 10 moles of air and occupying a volume of 40L loses half its volume due
to a puncture. Considering that the pressure and temperature remain constant, what
would be the amount of air in the deflated tyre?
Given,
The initial amount of air (n1) = 10 mol
The initial volume of the tyre (V1) = 40 L
The final volume of the tyre (V2) = 20 L
According to Avogadro’s law, the final amount of air in the tyre (n2) = (V2n1)/V1 = 5 moles.
The deflated tyre would contain 5 moles of air.
To learn more about Avogadro’s law and other important gas laws, such as Boyle’s law,
register with BYJU’S and download the mobile application on your smartphone.

10. A balloon has been filled to a volume of 1.90L1.90L with 0.0920mol0.0920mol of helium
gas. If 0.0210mol0.0210mol of additional helium is added to the balloon while the
temperature and pressure are held constant, what is the new volume of the balloon?
Given:
V1=1.90LV1=1.90L
n1=0.0920moln1=0.0920mol
Find: V2=?L

First, rearrange the equation algebraically to solve for V2V2.

V2=V1×n2n1(11.8.3)
Now substitute the known quantities into the equation and solve.

V2=1.90L×0.1130mol0.0920mol=2.33L(11.8.4)
Answer:
0.350 L
Rectilinear Motion

Rectilinear motion is another name for straight-line motion. This type of motion describes the
movement of a particle or a body.

A body is said to experience rectilinear motion if any two particles of the body travel the same
distance along two parallel straight lines. The figures below illustrate rectilinear motion for a
particle and body.

Rectilinear motion for a particle:

Rectilinear motion for a body:

In the above figures, x(t) represents the position of the particles along the direction of motion,
as a function of time t.

Given the position of the particles, x(t), we can calculate the displacement, velocity, and
acceleration. These are important quantities to consider when evaluating the kinematics of a
problem.

A common assumption, which applies to numerous problems involving rectilinear motion, is


that acceleration is constant. With acceleration as constant we can derive equations for the
position, displacement, and velocity of a particle, or body experiencing rectilinear motion.

The easiest way to derive these equations is by using Calculus.

The acceleration is given by

where a is the acceleration, which we define as constant.

Integrate the above equation with respect to time, to obtain velocity. This gives u
where v(t) is the velocity and C1 is a constant.

Integrate the above equation with respect to time, to obtain position. This gives u

where x(t) is the position and C2 is a constant.

The constants C1 and C2 are determined by the initial conditions at time t = 0. The initial
conditions are:

At time t = 0 the position is x1.

At time t = 0 the velocity is v1.

Substituting these two initial conditions into the above two equations we get
Equations (1), (2), (3), and (4) fully describe the motion of particles, or bodies experiencing rectilinear
(straight-line) motion, where acceleration a is constant.

For the cases where acceleration is not constant, new expressions have to be derived for the position,
displacement, and velocity of a particle. If the acceleration is known as a function of time, we can use
Calculus to find the position, displacement, and velocity, in the same manner as before.

Alternatively, if we are given the position x(t) as a function of time, we determine the velocity by
differentiating x(t) once, and we determine the acceleration by differentiating x(t) twice.

For example, let's say the position x(t) of a particle is given by


Examples 1:
Example 2
Example 3:
4.Find the integral expression that would result in the total distance traveled on the
interval [0, 3] if the velocity is given by v(t)=t2−4.

Solution.

To find the total distance we need to integrate the speed function, i.e. the absolute value
of the velocity. Note that the velocity changes sign at t=2. Therefore we split the
interval [0,3] into two intervals [0,2] and [2,3]. The total distance s traveled by the
particle on the interval [0,3] is expressed in the form:
s=2∫0∣∣t2−4∣∣dt+3∫2∣∣t2−4∣∣dt.
Given that the velocity is negative on the first interval and positive on the second
interval, we get

s=−2∫0(t2−4)dt+3∫2(t2−4)dt.
Rearranging the terms, we have

s=3∫2(t2−4)dt−2∫0(t2−4)dt.

5.A particle is moving along the x-axis so that its position at time t≥0 is given by the
equation x(t)=tlnt. Determine the acceleration of the particle when the velocity is
zero.

Solution.
Find the particle’s velocity by differentiating the position function:

v(t)=x′(t)=(tlnt)′=1⋅lnt+t⋅1t=lnt+1.
Continue differentiating to find the acceleration:

a(t)=v′(t)=(lnt+1)′=1t.
The velocity is zero when time is

v(t)=0,⇒lnt+1=0,⇒lnt=−1,⇒t=1e.
Substituting this time value, we find the acceleration at this instant:

a=11e=e.

6.When two particles start at the origin with velocities v(t)=cost and u(t)=sin2t, how
many times in the interval [0,2π] will their speeds be equal?

Solution

7.A particle moves along a straight line according to the


equation x(t)=t3−6t2+5, where x is in meters, t is in seconds. Find the total distance
traveled by the particle after 6 seconds.

Solution.
To find the total distance traveled by a particle, we need to take the integral of of the
speed |v(t)|:
s=t2∫t1|v(t)|dt
Determine the particle’s velocity:

v(t)=x′(t)=(t3−6t2+5)′=3t2−12t=3t(t−4).
We see that the velocity is negative for 0<t<4 and positive when t>4. Hence, we split up
the integral into the following two components:
s=6∫0|v(t)|dt=4∫0|v(t)|dt+6∫4|v(t)|dt.
Given that the velocity is negative in the first integral and positive in the second, we
obtain:

s=−4∫0v(t)dt+6∫4v(t)dt.
This yields:
s=−(t3−6t2+5)∣∣40+(t3−6t2+5)∣∣64=−[(64−96+5)−5]+[(216−216+5)−(64−96+5)]=32+32=6
4.
So, the total distance traveled by the particle is equal to 64m.

8. A projectile is fired upward from a 15.3 m cliff at a speed of 19.6 m/s and allowed
to fall into a valley below. The acceleration g due to Earth’s gravity is about 9.8 m/s2
, or about 32 ft/s 2 , downward. (a) Given that a(t) = –9.8 m/s2 , find v(t) and use it to
find the time at which the projectile reaches its maximum height. Find the maximum
height of the projectile using geometry. Solution: If a(t) = –9.8, then v(t) = –9.8t + C
for some constant C. Note that v(0) = C in this problem, so that C is the initial
velocity. Therefore, v(t) = –9.8t + 19.6 m/s. The maximum height occurs when the
velocity is zero, so –9.8t + 19.6 = 0 implies that the maximum height occurs at t = 2
seconds. Although we do not have a position function, we can find the maximum
height using geometry. Since the maximum height in this problem is simply the
displacement over the first 2 seconds, and the displacement is the net area bounded
by the velocity curve, we see that the maximum height is the area of the shaded
triangle, which is (1/2)(2)(19.6) = 19.6 meters. (b) Use geometry to find the
displacement and total distance traveled over the interval [0, 3]. Solution: The
displacement is the net area bounded by v(t), and the total distance traveled is the
total area. In this case, we can use the two triangles in the figure to compute
displacement on [0, 3] as (1/2)(2)(19.6) – (1/2)(1)(9.8) = 14.7 meters, and the total
distance traveled on [0, 3] as (1/2)(2)(19.6) + (1/2)(1)(9.8) = 24.5 meters.

(b) Use geometry to find the displacement and total distance traveled over the interval
[0, 3]. Solution: The displacement is the net area bounded by v(t), and the total distance
traveled is the total area. In this case, we can use the two triangles in the figure to
compute displacement on [0, 3] as (1/2)(2)(19.6) – (1/2)(1)(9.8) = 14.7 meters, and the
total distance traveled on [0, 3] as (1/2)(2)(19.6) + (1/2)(1)(9.8) = 24.5 meter
9.A car is driven along a straight track with position given by s(t) = 150t – 300 ft (t in
seconds). (a) Find v(t) and a(t). Solution: We are given that s(t) = 150t – 300 ft, so
v(t) = s'(t) = 150 ft/s, and a(t) = v'(t) = 0 ft/s2 . (b) Use calculus to find the
displacement and total distance traveled over the interval [1, 4]. Solution: The
displacement on [1, 4] is simply the definite integral of velocity on [1, 4]:
Displacement 150 (150 ) 150 4 150 1 450 4 1 4 1         dt t ft. Since the
velocity of the car is constant, the car is always moving in the same direction.
Therefore, the total distance traveled is the same as the displacement in this case.

10.The vertical position of a body under constant acceleration is given by 1 2 y = Yo


+ Vyot + 2 at , where t = time, y = vertical position at time t, VyO = initial velocity in
vertical direction, a = acceleration in vertical direction, yO = initial position. By the
use of calculus, find the velocity and acceleration of the body as a function of time.
Also construct a graph of velocity vs. time and acceleration vs. time for this case.
Curvilinear Motion

General Curvilinear Motion

Curvilinear motion is defined as motion that occurs when a particle travels along a curved path. The
curved path can be in two dimensions (in a plane), or in three dimensions. This type of motion is more
complex than rectilinear (straight-line) motion
Three-dimensional curvilinear motion describes the most general case of motion for a particle.

To find the velocity and acceleration of a particle experiencing curvilinear motion one only needs to
know the position of the particle as a function of time.

Let’s say we are given the position of a particle P in three-dimensional Cartesian (x,y,z) coordinates, with
respect to time, where

The velocity of the particle P is given by

The acceleration of the particle P is given by

As you can see, if we know the position of a particle as a function of time, it is a fairly simple exercise to
find the velocity and acceleration. You simply take the first derivative to find the velocity and the second
derivative to find the acceleration.

The magnitude of the velocity of particle P is given by


The magnitude of the acceleration of particle P is given by

Note that the direction of velocity of the particle P is always tangent to the curve (i.e. the path traveled,
denoted by the blue curve in the figure above). But the direction of acceleration is generally not tangent
to the curve.

However, the acceleration component tangent to the curve is equal to the time derivative of the
magnitude of velocity of the particle P (along the curve). In other words, if vt is the magnitude of the
particle velocity (tangent to the curve), the acceleration component of the particle tangent to the curve
(at) is simply

In addition, the acceleration component normal to the curve (an) is given by

where R is the radius of curvature of the curve at a given point on the curve (xp,yp,zp).

The figure below illustrates the acceleration components at and an at a given point on the curve
(xp,yp,zp).
For the specific case where the path of the blue curve is given by y = f(x) (two-dimensional motion), the
radius of curvature R is given by

where |x| means the “absolute value” of x. For example, |-2.5| = 2.5, and |3.1| = 3.1.

However, it is usually not necessary to know the radius of curvature R along a curve. But nonetheless, it
is informative to understand it on the basis of its relationship to the normal acceleration (an).

Curvilinear Motion In Polar Coordinates

It is sometimes convenient to express the planar (two-dimensional) motion of a particle in terms of


polar coordinates (R,θ), so that we can explicitly determine the velocity and acceleration of the particle
in the radial (R-direction) and circumferential (θ-direction). For this type of motion, a particle is only
allowed to move along the radial R-direction for a given angle θ.

For a particle P defined in polar coordinates (as shown below), we can derive a general equation for its
radial velocity (vr), radial acceleration (ar), circumferential velocity (vc), and circumferential acceleration
(ac).

Note that the circumferential direction is perpendicular to the radial direction.

The position of the particle P is given with respect to time, where


To find the velocity, take the first derivative of x(t) and y(t) with respect to time:

To find the acceleration, take the second derivative of x(t) and y(t) with respect to time:

Without loss of generality we can evaluate the velocities and accelerations at angle θ = 0, knowing that
(at this angle) radial velocity and radial acceleration is in the x-direction, and circumferential velocity and
circumferential acceleration is in the y-direction.

Setting θ = 0 we have:
Equations (1), (2), (3), and (4) fully describe the curvilinear motion of a particle P in polar coordinates.

The term dθ/dt is called angular velocity. It has units of rad/s. One rad (radian) = 57.296 degrees.

The term d2θ/dt2 is called angular acceleration. It has units of rad/s2.

Since vr and vc are perpendicular to each other, the magnitude of the velocity of particle P is given by

Since ar and ac are perpendicular to each other, the magnitude of the acceleration of particle P is given
by
Example Problem For Curvilinear Motion

A slotted link is rotating about fixed pivot O with a counterclockwise angular velocity of 3 rad/s, and a
clockwise angular acceleration of 2 rad/s2. The movement of the link is causing a rod to slide along the
curved channel, as shown. The radius of the channel as a function of θ is given by, R = 0.7θ (with R in
meters and θ in radians). Determine the velocity and acceleration components of the rod at θ = 45°

Solution

The angle θ = 45° is equal to π/4 radians. In the equations, counterclockwise angular velocity is positive,
and clockwise angular acceleration is negative (since it acts to “slow down” the rotational speed of the
link).

The radial velocity of the rod is given by equation (1):

(The radial velocity is in the direction of increasing R).

The circumferential velocity of the rod is given by equation (3):

(The circumferential velocity is in the direction of increasing θ).


The radial acceleration of the rod is given by equation (2):

(The radial acceleration is in the direction of decreasing R).

The circumferential acceleration of the rod is given by equation (4):

(The circumferential acceleration is in the direction of increasing θ).


Dynamics of Rotational Motion: Rotational Inertia

If you have ever spun a bike wheel or pushed a merry-go-round, you know that force is needed to
change angular velocity as seen. In fact, your intuition is reliable in predicting many of the factors that
are involved. For example, we know that a door opens slowly if we push too close to its hinges.
Furthermore, we know that the more massive the door, the more slowly it opens. The first example
implies that the farther the force is applied from the pivot, the greater the angular acceleration; another
implication is that angular acceleration is inversely proportional to mass. These relationships should
seem very similar to the familiar relationships among force, mass, and acceleration embodied in
Newton’s second law of motion. There are, in fact, precise rotational analogs to both force and mass.

Force is required to spin the bike wheel. The greater the force, the greater the angular acceleration
produced. The more massive the wheel, the smaller the angular acceleration. If you push on a spoke
closer to the axle, the angular acceleration will be smaller.
Making Connections: Rotational Motion Dynamics

Dynamics for rotational motion is completely analogous to linear or translational dynamics. Dynamics is
concerned with force and mass and their effects on motion. For rotational motion, we will find direct
analogs to force and mass that behave just as we would expect from our earlier experiences.

Rotational Inertia and Moment of Inertia

Before we can consider the rotation of anything other than a point mass like the one in [link], we must
extend the idea of rotational inertia to all types of objects. To expand our concept of rotational inertia,
we define the moment of inertia
where net is the total torque from all forces relative to a chosen axis. For simplicity, we will only
consider torques exerted by forces in the plane of the rotation. Such torques are either positive or
negative and add like ordinary numbers. The relationship in is the rotational analog
to Newton’s second law and is very generally applicable. This equation is actually valid for any torque,
applied to any object, relative to any axis.

As we might expect, the larger the torque is, the larger the angular acceleration is. For example, the
harder a child pushes on a merry-go-round, the faster it accelerates. Furthermore, the more massive a
merry-go-round, the slower it accelerates for the same torque. The basic relationship between moment
of inertia and angular acceleration is that the larger the moment of inertia, the smaller is the angular
acceleration. But there is an additional twist. The moment of inertia depends not only on the mass of an
object, but also on its distribution of mass relative to the axis around which it rotates. For example, it
will be much easier to accelerate a merry-go-round full of children if they stand close to its axis than if
they all stand at the outer edge. The mass is the same in both cases; but the moment of inertia is much
larger when the children are at the edge.
Examples:

1. A beam 140 cm in length. There are three forces acts on the beam, F1 = 20 N, F2 = 10 N, and F3 = 40 N
with direction and position as shown in the figure below. What is the torque causes the beam rotates
about the center of mass of the beam?

Known :
The center of mass located at the center of the beam.
Length of beam (l) = 140 cm = 1.4 meters
Force 1 (F1) = 20 N, the lever arm 1 (l1) = 70 cm = 0.7 meters
Force 2 (F2) = 10 N, the lever arm 2 (l2) = 100 cm – 70 cm = 30 cm = 0.3 meters
Force 3 (F3) = 40 N, the lever arm 3 (l3) = 70 cm = 0.7 meters
Wanted : The magnitude of torque
Solution :
The torque 1 rotates beam clockwise, so assigned a negative sign to the torque 1.
τ1 = F1 l1 = (20 N)(0.7 m) = -14 N m
The torque 2 rotates beam counterclockwise, so assigned a positive sign to the torque 2.
τ2 = F2 l2 = (10 N)(0.3 m) = 3 N m
The torque 3 rotates beam clockwise, so assigned a positive sign to the torque 3.
τ3 = F3 l3 = (40 N)(0.7 m) = -28 N m
The net torque :
Στ = -14 Nm + 3 Nm – 28 Nm = – 42 Nm + 3 Nm = -39 Nm
The magnitude of the torque is 39 N m. The direction of rotation of the beam clockwise, so assigned a
negative sign.

2. What is the net torque acts on the beam The axis of rotation at point D. (sin 53o = 0.8)
Known :

The axis of rotation at point D


F1 = 10 N and l1 = r1 sin θ = (40 cm)(sin 53o) = (0.4 m)(0.8) = 0.32 meters
F2 = 10√2 N and l2 = r2 sin θ = (20 cm)(sin 45o) = (0.2 m)(0.5√2) = 0.1√2 meters
F3 = 20 N and l3 = r1 sin θ = (10 cm)(sin 90o) = (0.1 m)(1) = 0.1 meters
Wanted : The net torque
Solution :
τ1 = F1 l1 = (10 N)(0.32 m) = 3.2 Nm
(The torque 1 rotates beam counterclockwise so we assign positive sign to the torque 1)
τ2 = F2 l2 = (10√2 N)( 0.1√2 m) = -2 Nm
(The torque 2 rotates beam clockwise so we assign negative sign to the torque 2)
τ3 = F2 l2 = (20 N)(0.1 m) = 2 Nm
(The torque 3 rotates beam counterclockwise so we assign positive sign to the torque 3)
The net torque :
Στ = τ1 – τ1 + τ3
Στ = 3.2 Nm – 2 Nm + 2 Nm
Στ = 3.2 Nm

3. What is the net torque if the axis of rotation at point D. (sin 53o = 0.8)
Known :

The axis of rotation at point D.


Distance between F1 and the axis of rotation (rAD) = 40 cm = 0.4 m
Distance between F2 and the axis of rotation (rBD) = 20 cm = 0.2 m
Distance between F3 and the axis of rotation (rCD) = 10 cm = 0.1 m
F1 = 10 Newton
F2 = 10√2 Newton
F3 = 20 Newton
Sin 53o = 0.8
Wanted : The net torque
Solution :
The moment of the force 1
Στ1 = (F1)(rAD sin 53o) = (10 N)(0.4 m)(0.8) = 3.2 N.m
(The torque 1 rotates beam counterclockwise so we assign positive sign to the torque 1)
The moment of the force 2
Στ2 = (F2)(rBD sin 45o) = (10√2 N)(0.2 m)(0.5√2) = -2 N.m
(The torque 2 rotates beam clockwise so we assign negative sign to the torque 2)
The moment of the force 3
Στ3 = (F3)(rCD sin 90o) = (20 N)(0.1 m)(1) = 2 N.m
(The torque 2 rotates beam counterclockwise so we assign positive sign to the torque 3)
The net torque :
Στ = Στ1 + Στ2 + Στ3
Στ = 3.2 – 2 + 2
Στ = 3.2 Newton meter
4. Length of wire = 12 m, l1 = 4 m. Ignore wire’s mass. What is the moment of inertia of the system.

Known :
Mass of A (mA) = 0.2 kg
Mass of B (mB) = 0.6 kg
Distance between A and the axis of rotation (rA) = 4 meters
Distance between B and the axis of rotation (rB) = 12 – 4 = 8 meters
Wanted : The moment of inertia of the system
Solution :
The moment of inertia of A
IA = (mA)(rA2) = (0.2)(4)2 = (0.2)(16) = 3.2 kg m2
The moment of inertia of B
IB = (mB)(rB2) = (0.6)(8)2 = (0.6)(64) = 38.4 kg m2
The moment of inertia of the system :
I = IA + IB = 3.2 + 38.4 = 41.6 kg m2

5. A 6-N force is applied to a cord wrapped around a pulley of mass M = 5 kg and radius R = 20 cm. What
is the angular acceleration of the pulley. The pulley is a uniform solid cylinder.
Known :
Force (F) = 6 Newton
Mass (M) = 5 kg
Radius (R) = 20 cm = 20/100 m = 0.2 m
Wanted : Angular acceleration (α)
Solution :
The moment of the force :
τ = F R = (6 Newton)(0.2 meters) = 1.2 N m
The moment of inertia for solid cylinder :
I = 1/2 M R2
I = 1/2 (5 kg)(0.2 m)2
I = 1/2 (5 kg)(0.04 m2)
I = 1/2 (0.2)
I = 0.1 kg m2.
The angular acceleration :
τ=Iα
α = τ / I = 1.2 / 0.1 = 12 rad s-2
6. A block of mass = 4 kg hanging from a cord wrapped around a pulley of mass = 8 kg and radius R = 10
cm. Acceleration due to gravity is 10 ms-2 . What is the linear acceleration of the block? The pulley is a
uniform solid cylinder.
Known :
Mass of pulley (m) = 8 kg
Radius of pulley (r) = 10 cm = 0.1 m
Mass of block (m) = 4 kg
Acceleration due to gravity (g) = 10 m/s2
Weight (w) = m g = (4 kg)(10 m/s2) = 40 kg m/s2 = 40 Newton
Wanted : The free fall acceleration of the block
Solution :
The moment of inertia of the solid cylinder :
I = 1/2 M R2 = 1/2 (8 kg)(0.1 m)2 = (4 kg)(0.01 m2) = 0.04 kg m2
The moment of the force :
τ = F r = (40 N)(0.1 m) = 4 Nm
The angular acceleration :
Στ = I α
4 = 0.04 α
α = 4 / 0.04 = 100
The linear acceleration :
a = r α = (0.1)(100) = 10 m/s2
7. A block with mass of m hanging from a cord wrapped around a pulley. If the free fall acceleration of
the block is a m/s2, what is the moment of inertia of the pulley..
Known :

weight = w = m g
Lever arm = R
The angular acceleration = α
The free fall acceleration of the block = a ms-2
Wanted: The moment of inertia of the pulley (I)
Solution :
The connection between the linear acceleration and the angular acceleration :
a=Rα
α=a/R
The moment of inertia :
τ=Iα
I = τ : α = τ : a / R = τ (R / a) = τ R a-1
8. A 0.2-gram particle moves in a circle at a constant speed of 10 m/s. The radius of the circle is 3 cm.
What is the angular momentum of the particle?
Known :
Mass of particle (m) = 0.2 gram = 2 x 10-4 kg
Angular speed (ω) = 10 rad s-1
Radius (r) = 3 cm = 3 x 10-2 meters
Wanted : The angular momentum of the particle
Solution :
The equation of the angular momentum :
L=Iω
I = the angular momentum, I = the moment of inertia, ω = the angular speed
The moment of inertia (for particle) :
I = m r2 = (2 x 10-4 )(3 x 10-2)2 = (2 x 10-4 )(9 x 10-4) = 18 x 10-8
The angular momentum :
L = I ω = (18 x 10-8)(10 rad s-1) = 18 x 10-7 kg m2 s-1
Plane Motion

Velocity refers to a physical vector quantity which is described by both magnitude and direction. The
magnitude or scalar absolute value of velocity is referred to as speed. As stated by the Pythagorean
Theorem, the magnitude of the velocity vector is given by –

| v | = v = √ ( vx ²+ vy ² )

Acceleration is defined by the rate of change of velocity of an object with respect to time. Numerically or in
terms of components, it can be presented as –

ax = ddt vx
ay = ddt vy

Motion in a Plane

Motion in a plane is also referred to as a motion in two dimensions. For example, circular motion, projectile
motion, etc. For the analysis of such type of motion, the reference point will be made of an origin and the
two coordinate axes X and Y.

Motion in a plane refers to the point where we consider motion in two dimensions as only two dimensions
makes a plane. Here, considering the above, we take two axes into consideration – generally X-axis or Y –
axes. In an attempt to derive the equation of the motion in a plane, we must know about motion in
one direction.

Equations of Plane Motion

The equations of motion in a straight line are:

v = u+at

s = ut+1/2 at²

v2 = u² + 2as

Where,

 v = final velocity of the particle

 u = initial velocity of the particle


 s = displacement of the particle

 a = acceleration of the particle

 t = the time interval in which the particle is in consideration


In a plane, we have to apply the same equations separately in both the directions: Y axis and Y-axis. This
would give us the equations for motion in a plane.

vy = uy + ayt

sy = uy t +1/2 ay t²

v²y = u²y+2ay s

Where,

 vy = final velocity of the particle in the y-direction

 uy = initial velocity of the particle in the y-direction

 sy = displacement of the particle in the y-direction

 ay = acceleration of the particle in the y-direction


Similarly, for the X-axis :

vx = ux + ax t

sx= ux t+1/2 axt²

v²x = u²x+2 axs

Where,

 vx = final velocity of the particle in the x-direction

 ux = initial velocity of the particle in the x-direction

 sx= displacement of the particle in the x-direction

 ax = acceleration of the particle in x-direction


Projectile Motion: Plane Motion

One of the most common examples of motion in a plane is Projectile motion. In a projectile motion, the only
acceleration acting is in the vertical direction which is acceleration due to gravity (g). Therefore, equations
of motion can be applied separately in the X-axis and Y-axis to find the unknown parameters.

The above diagram represents the motion of an object under the influence of gravity. It is an example of
projectile motion (an special case of motion in a plane).

Examples of Two-Dimensional Plane Motion

 Throwing a ball or a cannonball

 The motion of a billiard ball on the billiard table.

 A motion of a shell fired from a gun.

 A motion of a boat in a river.

 The motion of the earth around the sun.


Examples:

1.A stone is thrown from ground level over horizontal ground. It just clears three walls, the successive
distances between them being r and 2r. The inner wall is 15/7 times as high as the outer walls which are
equal in height. The total horizontal range is nr, where n is an integer. Find n.

Solution

Let us just assume that both the outer walls are equal in height say h and they are at equal distance x
from the end points of the parabolic trajectory as can be shown below in the figure.

Now equation of the parabola is


y=bx−cx2 (1)
y=0 t x=nr=R
where R is the range of the parabola.
Putting these values in equation (1) we get
b=cnr (2)
Now the range RR of the parabola is
R=a+r+2r+a=nr
This gives
a=(n−3)r/2 (3)
The trajectory of the stone passes through the top of the three walls whose coordinates are
(a,h),(a+r,15h/7),(a+3r,h)
Using these co-ordinates in equation 1 we get
h=ab−ca2 (4)
2
15h/7=b(a+r)−c(a+r) (5)
h=b(a+3r)−c(a+3r)2 (6)
After combining (2), (3), (4), (5) and (6) and solving them we get n = 4.
2.Which two quantities are constant throughout projectile motion when air resistance is negligible?
(A) The speed and acceleration.
(B) The vertical component of velocity and acceleration.
(C) The magnitude of the acceleration and the speed.
(D) The horizontal component of velocity and acceleration.
(E) The direction of the velocity and acceleration.

Solution:

(A) The speed and acceleration.


- speed is not constant. a = constant = g.
(B) The vertical component of velocity and acceleration.
- ay ≠ 0 ⇒ vy not constant.
(C) The magnitude of the acceleration and the speed.
- speed is not constant.
(D) The horizontal component of velocity and acceleration.
- ax = 0 ⇒ vx = constant.
(E) The direction of the velocity and acceleration.
- Direction of v changes.
Therefore answer is D.

3.A truck goes around a circular track of radius R at speed v so that it makes one circuit every T seconds
As it does so it experiences a centripetal acceleration of magnitude p
If the trcuk now goes around a different circular track of radius 4R so that it now takes a time ½T to go
around once, what is the magnitude of its centripetal acceleration?
(A) ½ p
(B) 16p.
(C) 4p.
(D) 8p.

Solution
Initial situation
a1 = v2 /R
Now v=2πR/T
or a1= 4π2 R/T2
On the second track:
a2= 4π2 (4R)/ (.5T)2
=64π2 R/T2
Now
a2 = 16 a1 =16c
4.A particle moves such that
x = (18.0)t
and y = 4t – 4.90t2

(a) Write a vector expression for the particle position as a function of time, using the unit vectors i and j
(b) Obtain the expression for the velocity vector as a function of time
(c) Obtain the expression for the acceleration vector a as a function of time.
(d) Find the the position, the velocity, and the acceleration of the particle at t = 1.00 s.

Solution

(a) Position vector is given by


r= xi + yj
Therefore
r= (18.0)t i + [4t – 4.90t2 ] j
(b) v= dr/dt =d[(18.0)t i + [4t – 4.90t2 ] j ]/dt =18 i + (4-9.8t)j
(c) a= dv/dt =-9.8 j
(d) At t=1 sec
r= (18.0)t i + [4t – 4.90t2 ] j
r= (18.0)i – .9 j
(e) At t = 100 s,
v=18 i + (4-9.8t)j =18i -5.8 j
(f)At 1= 3.00 s,
a = -9.8 j
5. A rabbit runs across a parking lot on which a set of coordinate axes has, strangely enough, been
drawn. The path is such that the components of the rabbit's position with respect to the origin of the
coordinate frame are given as function of time by

The units of the numerical coefficients in these equations are such that, if you substitute t in seconds, x
and y are in meters.

The position vector of the rabbit at time t can be expressed as:

From the equations of motion for x(t) and y(t) we can calculate the velocity and acceleration:

The acceleration of the rabbit is constant (independent of time).

The scalar product of the velocity and the acceleration will tell us something about the change of
velocity of the rabbit:

From this equation we conclude that for t < 14.6 s (negative scalar product) the rabbit will slow down,
while for t > 14.6 s (positive scalar product) the rabbits speed will increase. To check this prediction, we
calculate the magnitude of the velocity of the rabbit:

The velocity of the rabbit has a minimum at t = 14.6 s.


6. Suppose a projectile is launched with an initial velocity v0 and an angle [theta] with respect to the x-
axis (see Figure 4.2). What is its range R ?

We start with defining the coordinate system to be used in this problem:

x(t = 0) = x0 = 0

y(t = 0) = y0 = 0

The position of the projectile at any given time t can be obtained from the following expressions:

At impact, y(t) = 0. The time of impact can therefore be obtained by requiring that y(t) = 0 and solving
for t:

This equation has two solutions:

The first solution corresponds to the time that the projectile was launched, while the second solution
gives us the time that the projectile hits the ground again. The x-coordinate at that time can be obtained
by substituting the expression for t into the expression for x(t):

The maximum range is obtained when sin(2[theta]) = 1, which corresponds to [theta] = 45deg.. The
velocity of the projectile on impact can be calculated using the equations for vx(t) and vy(t):

Comparing the velocity on impact with the velocity at t = 0, we observe that the velocity component
parallel to the x-axis is unchanged, while the component along the y-axis changed sign.
If we look at the equation of the range R, we observe that the for each value of R (less than R max) there
are two possible launch angles: 45deg. + [Delta][theta] and 45deg. - [Delta][theta] (sin(2[theta]) is
symmetric around [theta] = 45deg.). The time of flight for the two cases are however different: a larger
launch angle corresponds to a longer time of flight (time of flight is proportional to sin([theta])).

7. An antitank gun is located on the edge of a plateau that is 60 m above the surrounding plain. The gun
crew sights an enemy tank stationary on the plain at a horizontal distance of 2.2 km from the gun. At the
same moment, the tank crew sees the gun and starts to move directly away from it with an acceleration
of 0.90 m/s2. If the antitank gun fires a shell with a muzzle speed of 240 m/s at an elevation of 10deg.
above the horizontal, how long should the gun crew wait before firing if they are to hit the tank.

Our starting point are the equations of motion of the shell.

(1)

(2)

Our coordinate system is defined such that the shell is launched at t = 0 s and its location at that instant
is specified by x = 0 m and y = h. Therefore, x0 = 0 m and y0 = h. In order to determine the trajectory of
the shell we first determine its time of flight between launch and impact. This time t1 can be obtained
from eq.(2) by requiring that on impact y(t1) = 0 m. Thus

(3)

The solutions for t1 are

(4)

Since the shell will hit the ground after being fired (at t = 0) we only need to consider the positive
solution for t1. The range R of the shell can be obtained by substituting t1 into eq.(1):

(5)

The problem provides the following information concerning the firing of the projectile:

h = 60 m

v0 = 240 m/s

[theta] = 10deg.
Using this information we can calculate v0x and v0y:

v0x = v0 cos([theta]) = 236 m/s

v0y = v0 sin([theta]) = 42 m/s

Substituting these values in eq.(4) and eq.(5) we obtain:

t1 = 9.8 s

R = 2320 m

The distance between the impact point and the original position of the tank is 120 m. The tank starts
from rest (v = 0 m/s) and has an acceleration of 0.9 m/s2. The time t2 it takes for the tank to travel to the
impact point can be found by solving the following equation:

(6)

This shows that

(7)

If the shell is fired at the same time that the tank starts to move, the tank will not reach the impact point
until (16.3 - 9.8) s = 6.5 s after the shell has landed. This means that the gun crew has to wait 6.5 s
before firing the antitank gun if they are to hit the tank.

8. Alex, parked by the side of an east-west road, is watching car P, which is moving in a westerly
direction. Barbara, driving east at a speed vBA = 52 km/h, watches the same car. Take the easterly
direction as positive.

a) If Alex measures a speed of 78 km/h for car P, what velocity will Barbara measure ?

This problem involves three cars that are located somewhere along a highway running from east to
west. Our coordinate system is chosen such that a positive velocity corresponds to motion towards the
east while a negative velocity corresponds to motion towards the west. Alex is sitting in a car parked
along the side of the highway (frame A) and Barbara is driving east with a velocity vBA (measured by
Alex) equal to + 52 km/h. Both observe a car P. The velocity of P, VPA (measured by Alex), is - 78 km/h
(see Figure 4.6). The velocity of car P measured by Barbara can be calculated as follows:

vPA = vBA + vPB


vPA = - 78 km/h

vBA = 52 km/h

vPB = vPA - vBA = (- 78 - 52) km/h = - 130 km/h

The velocity of Alex measured by Barbara can be calculated in a similar manner, and the resulting
velocity is - 52 km/h. The corresponding velocity diagram is shown in Figure 4.7.

Figure 4.6. Velocity Diagram Measured Relative to Alex.

Figure 4.7. Velocity Diagram Measured Relative to Barbara.

b) If ALEX sees car P brake to a halt in 10 s, what acceleration (assumed constant) will he measure for it ?

Alex observes that car P brakes to a halt in 10 s. Alex can calculate the acceleration of car P:

v(t) = v0 + a t

The initial conditions are:

v0 = v(t = 0) = - 78 km/h = - 21.7 m/s

The final conditions are:

v(t = 10 s) = 0 m/s

Assuming a constant acceleration, a can be calculated as follows:


c) What acceleration would Barbara measure for the braking car ?

Barbara also observe that car P slows down during this 10 s interval. At time t = 0 she observes car P
moving with a velocity equal to v0 = - 130 km/h (- 36.1 m/s). After 10 s, she observes car P moving with a
velocity equal to v(t = 10 s) = - 52 km/h (- 14.4 m/s). Barbara can also calculate the acceleration of car P:

Both Alex and Barbara measure the same acceleration of car P. This is what we expected since Barbara
moves with a constant acceleration with respect to Alex.

9. Cars cruise down an expressway at 25 m/s. Engineers are designing an off-ramp in an interchange
with a deceleration of −2.0 m/s2 that lasts 3.0 s.

a. What velocity will cars have at the end of the off-ramp?


b. What minimum length should the ramp have?
c. What maximum velocity could a car entering the off-ramp have and still be able to exit at the
intended velocity? (Assume an extreme deceleration of four times the usual rate.)

Solution:

a. State the givens and the unknown. Use the first equation of motion — the one where velocity is
a function of time.

v0 = 25 m/s
a = −2.0 m/s2
Δt = 3.0 s
v=?
v = v0 + at
v = (25 m/s) + (−2.0 m/s2)(3.0 s)
v = 19 m/s
c. Restate the givens and the unknown from the previous part, since they're all still valid. Use the
second equation of motion — the one where displacement is a function of time.

v0 = 25 m/s
a = −2.0 m/s2
Δt = 3.0 s
Δs = ?
Δs = v0t + ½at2
Δs = (25 m/s)(3.0 s) + ½(−2.0 m/s2)(3.0 s)2
Δs = 66 m
e. The final velocity calculated in part a. is still the final velocity. (All cars need to exit at the same
speed.) The displacement calculated in part b. is still the displacement. (All cars get the same amount of
space to slow down.) The new acceleration is four times the old one. The new initial velocity is the new
unknown. No time is given or can be inferred. None is needed. Use the third equation of motion — the
one where velocity is a function of displacement and time is not a part of the equation.

v = 19 m/s
a = −8.0 m/s2
Δs = 66 m
v0 = ?
2 2
v = v0 + 2aΔs
v02 = v2 − 2aΔs
v02 = (19 m/s)2 − 2(−8.0 m/s2)(66 m)
2
v0 = 38 m/s

10. A typical commercial jet airliner needs to reach a speed of 180 knots before it can take off. (A knot is
a nautical mile per hour and is nearly equal to half a meter per second.) If such a plane spends 30 s on
the runway estimate…

a. its acceleration.
b. the minimum runway length.
To determine acceleration, I recommend using the definition of acceleration.

List the givens and unknown first.

v0 = 0 m/s
v = 180 kts ≈ 90 m/s
t = 30 s
a=?

Then use the equation.

Δv
a=
Δt
90 m/s
a=
30 s
a = 3 m/s2 ≈ ⅓ g
There are two ways to determine the runway length. Either method yields the same solution. Let's list
the givens and unknown first.

v0 = 0 m/s
v = 180 kts ≈ 90 m/s
t = 30 s
a = 3 m/s2
∆s = ?

Here's the solution using the distance-time equation.

s = s0 + v0t + ½at2
∆s = ½(3 m/s2)(30 s)2
∆s = 1350 m

And here's the solution using the average velocity equation.

∆s = vt = ½(v + v0)t
∆s = ½(90 m/s + 0 m/s)(30 m/s)
∆s = 1350 m
Work and energy

Whenever a force is applied to an object, causing the object to move, work is done by the force. If a
force is applied but the object doesn't move, no work is done; if a force is applied and the object moves
a distance d in a direction other than the direction of the force, less work is done than if the object
moves a distance d in the direction of the applied force.

The physics definition of "work" is:

The unit of work is the unit of energy, the joule (J). 1 J = 1 N m.

Work can be either positive or negative: if the force has a component in the same direction as the
displacement of the object, the force is doing positive work. If the force has a component in the
direction opposite to the displacement, the force does negative work.

If you pick a book off the floor and put it on a table, for example, you're doing positive work on the
book, because you supplied an upward force and the book went up. If you pick the book up and place it
gently back on the floor again, though, you're doing negative work, because the book is going down but
you're exerting an upward force, acting against gravity. If you move the book at constant speed
horizontally, you don't do any work on it, despite the fact that you have to exert an upward force to
counter-act gravity.

Kinetic energy

An object has kinetic energy if it has mass and if it is moving. It is energy associated with a moving
object, in other words. For an object traveling at a speed v and with a mass m, the kinetic energy is given
by:

The work-energy principle

There is a strong connection between work and energy, in a sense that when there is a net force doing
work on an object, the object's kinetic energy will change by an amount equal to the work done:

Note that the work in this equation is the work done by the net force, rather than the work done by an
individual force.

Gravitational potential energy

Let's say you're dropping a ball from a certain height, and you'd like to know how fast it's traveling the
instant it hits the ground. You could apply the projectile motion equations, or you could think of the
situation in terms of energy (actually, one of the projectile motion equations is really an energy equation
in disguise).

If you drop an object it falls down, picking up speed along the way. This means there must be a net force
on the object, doing work. This force is the force of gravity, with a magnitude equal to mg, the weight of
the object. The work done by the force of gravity is the force multiplied by the distance, so if the object
drops a distance h, gravity does work on the object equal to the force multiplied by the height lost,
which is:

work done by gravity = W = mgh (h = height lost by the object)

An alternate way of looking at this is to call this the gravitational potential energy. An object with
potential energy has the potential to do work. In the case of gravitational potential energy, the object
has the potential to do work because of where it is, at a certain height above the ground, or at least
above something.

Spring potential energy

Energy can also be stored in a stretched or compressed spring. An ideal spring is one in which the
amount the spring stretches or compresses is proportional to the applied force. This linear relationship
between the force and the displacement is known as Hooke's law. For a spring this can be written:

F = kx, where k is known as the spring constant.

k is a measure of how difficult it is to stretch a spring. The larger k is, the stiffer the spring is and the
harder the spring is to stretch.

If an object applies a force to a spring, the spring applies an equal and opposite force to the object.
Therefore:

force applied by a spring : F = - kx

where x is the amount the spring is stretched. This is a restoring force, because when the spring is
stretched, the force exerted by by the spring is opposite to the direction it is stretched. This accounts for
the oscillating motion of a mass on a spring. If a mass hanging down from a spring is pulled down and let
go, the spring exerts an upward force on the mass, moving it back to the equilibrium position, and then
beyond. This compresses the spring, so the spring exerts a downward force on the mass, stopping it, and
then moving it back to the equilibrium and beyond, at which point the cycle repeats. This kind of motion
is known as simple harmonic motion, which we'll come back to later in the course.

The potential energy stored in a spring is given by:

where x is the difference between the spring's length and its unstrained length.
In a perfect spring, no energy is lost; the energy is simply transferred back and forth between the kinetic
energy of the mass on the spring and the potential energy of the spring (gravitational PE might be
involved, too).

Conservation of energy

We'll take all of the different kinds of energy we know about, and even all the other ones we don't, and
relate them through one of the fundamental laws of the universe.

The law of conservation of energy states that energy can not be created or destroyed, it can merely be
changed from one form of energy to another. Energy often ends up as heat, which is thermal energy
(kinetic energy, really) of atoms and molecules. Kinetic friction, for example, generally turns energy into
heat, and although we associate kinetic friction with energy loss, it really is just a way of transforming
kinetic energy into thermal energy.

The law of conservation of energy applies always, everywhere, in any situation. There is another
conservation idea associated with energy which does not apply as generally, and is therefore called a
principle rather than a law. This is the principle of the conservation of mechanical energy.
Example:

1. A 5000-kg car accelerated from rest to 20 m/s. Determine the net work done on the car.
Known :
Mass (m) = 5000 kg
Initial speed (vo) = 0 m/s (car rest)
Final speed (vt) = 20 m/s
Wanted : net work
Solution :

The work-kinetic energy principle :


Wnet = ΔEK
Wnet = ½ m (vt2 – vo2)
Wnet = net work
ΔEK = the change in kinetik energy
m = mass (kg),
vt = final speed (m/s),
vo = initial speed (m/s).
Net work :
Wnet = ½ m (vt2 – vo2)
Wnet = ½ (5000)(202 – 02)
Wnet = (2500)(400 – 0)
Wnet = (2500)(400)
Wnet = 1000,000 Joule

2. A 10-kg object accelerated from 5 m/s to 10 m/s. Determine the net work done on the object!
Known :
Mass (m) = 10 kg
Initial speed (vo) = 5 m/s
Final speed (vt) = 10 m/s
Wanted : net work
Solution :
Net work :
Wnet = ΔEK
Wnet = ½ m (vt2 – vo2)
Wnet = ½ (10)(102 – 52)
Wnet = (5)(100 – 25)
Wnet = (5)(75)
Wnet = 375 Joule
3. A 2000-kg car decelerated from 10 m/s to 5 m/s. What is the work done on the car ?
Known :
Car’s mass (m) = 2000 kg
Initial speed (vo) = 10 m/s
Final speed (vt) = 5 m/s
Wanted: net work
Solution :

Net work :
Wnet = ΔEK
Wnet = ½ m (vt2 – vo2)
Wnet = ½ (2000)(52 – 102)
Wnet = (1000)(25 – 100)
Wnet = (1000)(-75)
Wnet = -75,000 Joule
The minus sign indicates that the direction of displacement is opposite with the direction of the net
force.

4. A 60-N constant force exerted on a 10-kg object for 12 seconds. The initial velocity of an object is 6
m/s and the direction of the object is the same as the direction of the force.

(1) Work done on the object is 30,240 Joule

(2) The final kinetic energy is 30,240 joule

(3) Power is 2,520 Watt

(4) Th increase in the kinetic energy of the object is 180 Joule

The correct statements are…

Known :

Force (F) = 60 N

Time interval (t) = 12 seconds

Mass of object (m) = 10 kg

Initial velocity (vo) = 6 m/s

Wanted : The correct statements

Solution :

Acceleration of object :

∑F = m a
60 = 10 a

a = 60 / 10 = 6 m/s2

The final velocity :

vt = vo + a t
vt = 6 + (6)(12)
vt = 6 + 72
vt = 78 m/s
The distance traveled in 12 seconds :
s = vo t + 1/2 a t2
s = (6)(12) + 1/2 (6)(12)2

s = 72 + (3)(144)

s = 72 + 432

s = 504 meters

(1) Work done by force

W = F s = (60)(504) = 30,240 Joule

(2) The final kinetic energy

KE = 1/2 m vt2 = 1/2 (10)(78)2 = (5)(6084) = 30,420 Joule

(3) Power

P = W / t = 30,240 / 12 = 2,520 Joule/second

(4) The increase in the kinetic energy

ΔKE = 1/2 m vt2 – 1/2 m vo2 = 1/2 m (vt2 – vo2) = 1/2 (10)(782 – 62) = 5 (6084 –36) = 5 (6048)

ΔKE = 30,240 Joule


5. The larger work is done by object number…

Solution :

Net work = change of th kinetic energy

Wnet = ½ m (vt2 – vo2)

The larger work :

W1 = ½ (8)(42 – 22) = (4)(16 – 4) = (4)(12) = 48 Joule


W2 = ½ (8)(52 –32) = (4)(25 – 9) = (4)(16) = 64 Joule
W3 = ½ (10)(62 – 52) = (5)(36 – 25) = (5)(11) = 55 Joule
W4 = ½ (10)(42 – 02) = (5)(16 – 0) = (5)(16) = 80 Joule
W5 = ½ (20)(32 – 32) = (10)(9 – 9) = (10)(0) = 0 Joule

6. A 4000-kg car travels along straight line at 25 m/s. The car is decelerated so that the car’s final velocity is 15 m/s.
What is the work done on the car.

Known :

Mass (m) = 4000 kg

The initial velocity (vo) = 25 m/s


The final velocity (vt) = 15 m/s

Wanted : Work done on car

Solution :

Wnet = ½ m (vt2 – vo2) = ½(4000)(152-252) = (2000)(225-625) = (2000)(-400) = -800,000 Joule = -800 kJ


7. A 0.1-kg thrown horizontally at 6 m/s from the height of 5 meters. If the acceleration of gravity is 10 m/s2, then
what is the kinetic energy of ball at the height of 2 meters.

Known :

Mass (m) = 0.1 kg

The change in height (h) = 5 m – 2 m = 3 meters

Acceleration due to gravity (g) = 10 m/s2

Wanted : The kinetic energy at the height of 2 meters.

Solution :

Projectile motion can be understood by analyzing the horizontal and vertical components of the motion
separately. Motion in horizontal direction analyzed as the constant velocity motion and motion in vertical direction
analyzed as free fall motion or vertical motion.

The initial mechanical energy = the gravitational potential energy.

PE = m g h = (0.1)(10)(3) = 3 Joule.

The final mechanical energy = the kinetic energy.

KE = 3 Joule.

8. A 1000-kg car accelerated from rest and travels at 5 m/s. What is the work done by car?

Known :

Mass (m) = 1000 kg

Initial velocity (vo) = 0


Final velocity (vt) = 5 m/s

Wanted : Work (W) done by car

Solution :

Wnet = ½ m (vt2 – vo2)

Work done by car :

Wnet = ½ (1000)(52 – 02) = (500)(25 – 0) = (500)(25) = 12,500 Joule


9. A 500-gram ball thrown vertical upward from the surface of earth with the initial velocity 10 m/s2. Acceleration
due to gravity is 10 ms-2. What is th work done by the weight force when ball reaches the maximum height.

Known :

Mass of ball (m) = 500 gram = 0.5 kg

Initial velocity (vo) = 10 m/s2


Final velocity (vt) = 0 (velocity at the highest point)
Acceleration due to gravity (g) = 10 m/s2

Wanted : Work (W) don by weight

Solution :

The net work done by net force on an object = the change in the kinetic energy.

Wnet = ΔEK = EKt – EKo


Wnet = ½ m vt2 – ½ m vo2 = ½ m (vt2 – vo2)
KEt = the final kinetic energy, KEo = the initial kinetic energy, m = mass of object, vt = the final velocity of object, vo =
initial velocity of object.

Net work :

Wnet = ½ m (vt2 – vo2) = ½ (0.5)(02 – 102)


Wnet = (0.25)(-100) = -25 Joule

Minus sign indicates that the direction of displacement is opposite to the weight of the ball. The direction of ball is
upright and the direction of weight is downright.
10. A 1-kg object free fall with the height difference = 2.5 meters. Acceleration due to gravity is 10 m.s -2. What is
the work done on the object?

Known :

Mass of ball (m) = 1 kg

Initial velocity (vo) = 0 m/s

Height (h) = 2.5 meters

Acceleration due to gravity (g) = 10 m/s2

Wanted : Net work during displacement

Solution :

Final velocity of ball (vt)


Calculated using the equation of free fall motion. Known : Acceleration due to gravity (g) = 10 m/s2, The change in
height of ball (h) = 2.5 meters. Wanted : Final velocity.
vt2 = 2 g h = 2(10)(2.5) = 2(25)
vt = √2(25)
vt = 5√2

Net work = the change in kinetic energy

Wnet = ΔEK = ½ m (vt2 – vo2) = ½ (1){(5√2)2 – 02}


Wnet = ½ (25)(2) = 25 Joule
Impulse and Momentum

When a net force acts on a body it will result in an acceleration which alters the motion of the body. A
large net force will cause a larger acceleration than a small net force. The total change in motion of the
object can be the same if the large and small forces act for different time intervals. The combination of
the force and time that it acts is a useful quantity which leads us to define impulse.

Impulse
Impulse is the product of the net force and the time interval for which the force acts.

Impulse=F⃗ net⋅Δt

However, from Newton's Second Law, we know that

F⃗ net∴F⃗ net⋅Δt=Δp⃗ Δt=Δp⃗ =ImpulseF→net=Δp→Δt∴F→net·Δt=Δp→=Impulse

Therefore we can define the impulse-momentum theorem:

Impulse=Δp⃗ Impulse=Δp→

Impulse is equal to the change in momentum of an object. From this equation we see, that for a given
change in momentum, F⃗ netΔtF→netΔt is fixed. Thus, if F⃗ netF→net is reduced, ΔtΔt must be increased
(i.e. a smaller resultant force must be applied for longer to bring about the same change in momentum).
Alternatively if ΔtΔt is reduced (i.e. the resultant force is applied for a shorter period) then the resultant
force must be increased to bring about the same change in momentum.
The graphs below show how the force acting on a body changes with time.

The area under the graph, shaded in, represents the impulse of the body.
Examples:

1. An object travels with a velocity 4m/s to the east. Then, its direction of motion and magnitude of
velocity are changed. Picture given below shows the directions and magnitudes of velocities. Find the
impulse given to this object.

I=F.Δt=Δp=m.ΔV
where ΔV=V2-V1=-3-4=-7m/s
I=m.ΔV=3.(-7)=-21kg.m/s

2. Ball having mass 4kg and velocity 8m/s travels to the east. Impulse given at point O, makes it change
direction to north with velocity 6m/s. Find the given impulse and change in the momentum.

Initial and final momentum vectors of ball are shown in the figure below.

P1=m.V1=4kg.8m/s=32kg.m/s
P2=m.V2=4kg.6m/s=24kg.m/s
ΔP=P2+P1 (vector addition)
ΔP2=P22+P12=m2(v22+v12)
ΔP2=16.100
ΔP=40kg.m/s
Impulse=change in momentum
I=ΔP=40kg.m/s

3. Find the impulse and force which make 12m/s change in the velocity of object having 16kg mass in 4
s.
F.Δt=ΔP=m.ΔV
F.4s=16kg.12m/s
F=48N
F.Δt=Impulse=192kg.m/s

4. Applied force vs. time graph of object is given below. Find the impulse of the object between 0-10s.

Area under the force vs. time graph gives us impulse.


F.Δt=20.2/2+20.(6-2)+20.(10-6)/2
F.Δt=140kg.m/s

5. A ball having mass 500g hits wall with a10m/s velocity. Wall applies 4000 N force to the ball and it
turns back with 8m/s velocity. Find the time of ball-wall contact.

F.Δt=ΔP=m.ΔV=m.(V2-V1)
-4000.Δt=0,5kg.(8-10)
Δt=0,00025s

6.Now, the bears I live with average, the males, eight to twelve hundred pounds [360 to 540 kg]. They're
the largest bears in the world…. They've been clocked at 41 [mph] and they've run a hundred meter
dash in 5.85 seconds, which a human on steroids doesn't even approach.

a. Compute the speed of a grizzly bear using Mr. Treadwell's hundred meter statement.
b. Compute the momentum of a grizzly bear using the speed you calculated in part a. and the
average mass stated by Mr. Treadwell.
c. How fast would a 250 lb man have to run to have the same momentum you calculated in part b?
(Do not use a calculator to compute your answer.)
d. How fast would a 4000 lb car have to drive to have the same momentum you calculated in part
b? (Do not use a calculator to compute your answer.)
solution

a. Speed is distance over time.

∆s
v=
∆t

100 m
v=
5.85 s

v = 17 m/s

b. Momentum is mass times velocity. Let's use a mass in the middle of the range stated by Mr.
Treadwell.

p = mv
p = (450 kg)(17 m/s)
p = 7700 kg m/s

c. Momentum is the product of mass and velocity, which makes the two quantities inversely
proportional. Mass goes down when we replace the 1,000 pound grizzly bear with a 250 pound
man. To keep the momentum constant, the man will have to run faster — faster by an amount
that is inversely proportional to the decrease in weight. Since our hypothetical man has ¼ the
mass of a grizzly, he needs to run 4 times faster to have the same momentum. With numbers
this simple, you should be able to compute the answers without a calculator.

p = mv ⇒ p = (¼m)(4v)

That's 68 m/s in International units or 164 mph in Anglo-American units. When it comes to momentum,
a bear at top speed is like a man running as fast as a race car.

vman = 4vbear

vman = 4(17 m/s) = 68 m/s

vman = 4(41 mph) = 164 mph

d. Use reasoning similar to part c. Mass goes up when we replace the 1,000 pound grizzly bear with
a 4,000 pound car. Four times the mass needs ¼ the speed to have the same momentum. Once
again, the numbers are simple.

p = mv ⇒ p = (4m)(¼v)

That's 4.25 m/s or 10.25 mph. A bear at top speed is like a car driving through a parking lot.

vcar = ¼vbear

vcar = ¼(17 m/s) = 4.25 m/s


vcar = ¼(41 mph) = 10.25 mph

7. 1. A small ball is thrown horizontally with a constant speed of 10 m/s. The ball hits the wall and
reflected with the same speed. What is the change in linear momentum of the ball?
Known :
Mass (m) = 0.2 kg
Initial speed (vo) = -10 m/s
Final speed (vt) = 10 m/s

The plus and minus sign indicates that the objects moves in opposite direction.

Wanted : the change in linear momentum (Δp)


Solution :
Formula of the change in linear momentum :
Δp = m vt – m vo = m (vt – vo)
The change in linear momentum :
Δp = 0.2 (10 – (-10)) = 0.2 (10 + 10)
Δp = 0.2 (20)
Δp = 4 kg m/s

8. A 10-gram ball falls freely from a height, hits the floor at 15 m/s, then reflected upward at 10 m/s.
Determine the impulse!
Known :
Mass (m) = 10 gram = 0.01 kg
Initial velocity (vo) = -15 m/s
Final velocity (vt) = 10 m/s
Wanted : Impulse (I)
Solution :
The impulse (I) equals the change in momentum (Δp)
I = m vt – m vo = m (vt – vo)
Impulse :
I = 0.01 (10 – (-15)) = 0.01 (10 + 15)
I = 0.01 (25)
I = 0.25 kg m/s
9.A 200-gram ball thrown horizontally with a speed of 4 m/s, then the ball was hit in the same
direction. The duration of the ball in contact with the bat is 2 milliseconds and the ball speed after
leaving the bat is 12 m/s. The magnitude of force exerted by the batter on the ball is …
Known :
Mass (m) = 200 gram = 0.2 kg
Initial velocity (vo) = 4 m/s
Final velocity (vt) = 12 m/s
Time interval (t) = 2 milliseconds = (2/1000) seconds = 0.002 seconds
Wanted : The magnitude of the force (F)
Solution :
Formula of impulse :
I=Ft
Formula of the change in momentum :
m vt – m vo = m (vt – vo)
The impulse (I) equals the change in momentum (Δp)
I = Δp
F t = m (vt – vo)
F (0.002) = (0.2)(12 – 4)
F (0.002) = (0.2)(8)
F (0.002) = 1.6
F = 1.6 / 0.002
F = 800 Newton

Potrebbero piacerti anche